Final Endocrinology 2024

Download as pdf or txt
Download as pdf or txt
You are on page 1of 143

ADR WORKBOOK SERIES - ENDOCRINOLOGY

ENDOCRINOLOGY

Dr. Rajesh Gubba


MD – General Medicine
DNB-Cardiology
ADR WORKBOOK SERIES - ENDOCRINOLOGY
MASTER OF ENDOCRINE ORCHESTRA

pituitary Gland Hypophysis cerebri


Level of which hormone is likely to increase after hypothalamic ablation?
1. A Growth hormone
2. Prolactin
3. FSH
4. ACTH

HAT
ANT. PITUITARY HORMONES

GAP LEFT Hypothal Infundi


G GH GIRA butam
A
ACTH CRH
MITE P Dopamine
prolactin
L
LH GnRH
E

F
Endorphins
FSH Dopamine
T
TSH TRH

No prolactin Anfituitary
prolactin
Posterior Pituitary Hormones: -
stored in post pituitary
oxytocin
vasopressin ADH
1
ADR WORKBOOK SERIES - ENDOCRINOLOGY
Other Hormones in the Body

IGF I somatomedins
GH

Bone soft tissue

AC

Adrenalalf
A Medulla
A Cortex

Iii catecholamine

Z Fasciculata Z Reticularis E WE
Z Glomerulosa
weak sex Dopamine
hormones
steroids DHEA
Aldosterone
Androstenedione
RAAS

451M RTA Head injury ED CT

hemorrhage pituitary
An A except i steroids
2 ii DHEA
f Aldosterone in Andro
ADR WORKBOOK SERIES - ENDOCRINOLOGY
Other Hormones in the Body

prolactin

e spermatogenesis t Lactation
Testosterone ovulation

Estrogen 9
progesterone

F5 6

4 Estrogen a
4 Testosterone
progesterone
spermatogenesis
4 ovulation
3 corpus
luteum
ADR WORKBOOK SERIES - ENDOCRINOLOGY
Other Hormones in the Body

TSH GO
Released Active
Hormones

pen chief cells


Vit D PCI
bulars
calcitriol if HPL

calcitonin parafollicular Inhibin


renin ea
Thyroid Estro prog
Receptors for Endocrine Hormones: -
Melatonin

pineal GI
Cytoplasm(R)  Good Morning
circadian
G Glucocorticoids
M
rhythms
MC
Nuclear (R)  PET BOOKO
P
rogesterone
E
strogen Mistake write
T
estosterone
T
374
4
Revision
ADR WORKBOOK SERIES - ENDOCRINOLOGY
Membrane Receptors

 GPCRs: - KID
 K+ channel opening 
somatostatin


I
IP3/DAG/Ca+2 

Increase cAMP 
GnRH TRH oxytocin V R

V2 R
 Decrease cAMP 
Dopamine

Tyrosine kinase: - GIP pituitary


G

H
• I nsulin

• P
rolactin DYOsactomeg.lyDHpfacinod
Diabetes insipidus Hypopituitain

a) Definition: -
characterised by Red urination
31 24hr5 01
polyuria
40 50mikg in 24h
1) Central- s
ADH
R Resistent DI Pip
2) Nephrogenic -
ADH Normal D 1 conns
Physiology of ADH: -
d
DM 1kt no
psychogenic Diuretic
5
Osmo R abuse
Hypothalamus polydipsia
t
Thirst
d.es
ese
ADR WORKBOOK SERIES - ENDOCRINOLOGY

a synthesis
tev ADI

Reabs of H2O
A H2o consumption
Distal
tubules

Fluid volume sv.org Normal

VASOPRESSIN RECEPTORS

Vi R R I V2 R R2 V3 R

Ant
wall of B vessel
Endothelial Distal pituitar
tubules to
B vessel
vasoconstriction
y kidney ACTH
Hypovolemic shock VMF
f VIII
Abs H2O

Bleeding E varices B
6 B
Hemophilia D I
UN dis Nocturnal
V Oct 1
enuresis
ADR WORKBOOK SERIES - ENDOCRINOLOGY
Choose the best Lab value for a patient with central diabetes insipidus - Urinary
Osmolality & Serum Osmolality

1. 50 - 300
2. 500 - 260
3. 50 – 260
4. 500 - 100

Clinical Features of Diabetes Insipidus: -

polyuria Sr osm polydipsia


285 295
or Osm 300 9004
All of the following conditions are known to cause diabetes insipidus, except -
1. Multiple sclerosis
2. Head injury
3. Histiocytosis
4. Viral encephalitis

ETIOLOGY
• Pituitary form of DI: -
A) Acquired:
Central
• Head trauma (closed and penetrating)
B) Neoplasms-
1. Craniopharyngioma
2. Pituitary adenoma (suprasellar)
C) Granulomas-
1. Neurosarcoid
2. Histiocytosis
• Vascular - Sheehan's syndrome
Infections - Chronic meningitis, Viral encephalitis

B) Genetic:
1. Autosomal dominant (AVP-neurophysin gene)
2. Autosomal recessive (AVP-neurophysin gene)

7
ADR WORKBOOK SERIES - ENDOCRINOLOGY

Etiology of Nephrogenic D.I: -


Acquired:
1. Drugs-
Lithium
Demeclocylin
Methoxyflurane
Amphotericin B
Aminoglycosides
Cisplatin
2. Metabolic - Hypercalcemia, hypercalciuria
3. Infiltration- Amyloidosis
4. Vascular - Sickle cell disease

Genetic:
1. X-linked recessive (AVP receptor-2 gene)
2. Autosomal recessive (aquaporin-2 gene)
3. Autosomal dominant (aquaporin-2 gene)
• Wolfram syndrome, also called DIDMOAD (diabetes insipidus,
diabetes mellitus, optic atrophy, and deafness), is a rare autosomal-
recessive genetic disorder that causes childhood-onset diabetes
mellitus, optic atrophy, and deafness as well as various other possible
disorders.

A 33-year-old lady presents with polydipsia and polyuria. Her symptoms


started soon after a road traffic accident 6 months ago. The blood pressure is
120/80 mm Hg with no postural drop. The daily urinary output is 6- 8 liters.

I
Investigation showed, Na 130 mEq/L, K.3.5 mEq/L, urea 15mg/dL, sugar- 65
mg/dL. The plasma osmolality is 268 mosmol/L and urine osmolatity 45
mosmol/L. The most likely diagnosis is -
1. Central diabetes insipidus
2. Nephrogenic diabetes insipidus H2o Srosmf
3. Resolving acute tubular necrosis
consumption
4. Psychogenic polydipsia
smt apf
urosm
Which of the following statements about Diabetes Insipidus is true -

1.
2.
Urine osmolality should be >300 mosm/L
Plasma osmolality should be <280 mmol/L tied H2oreab
3. Water deprivation test is required
4. Plasma osmolality < Urine osmolality
U 09
HYO
so
osmdsr.ME 8
v Osm

Uriosm 10C water deprivation test


des
ADR WORKBOOK SERIES - ENDOCRINOLOGY

To differentiate Water deprivation test


polyurie by
CDI from NDI from With hold fluid Uriosm I
p.PT urosm
Increased urine Osmolarity Urine remains dilute
Normal Neurogenic D.I.
Svosma Psychogenic polydipsia Nephrogenic D.I.

AD AHzoveabs
Administer ADH Administer ADH

ur osm
Increased urine Urine Urine
Increased urine
osmolarity remains
osmolarity osmolarity
increased dilute
(It is less than 10% (Less than 10%
of higher than the increase in urine
Neurogenic Nephrogenic
maximal urine osmolarity
Diabetes insipidus
osmolarity than inspidius
achieved by water
restriction alone)

Treatment of Central DI DOC vasopressin analogue

Desmopressin Drugs ADH chlorpropamide

clotibrate

V10 UCR
v
carbamazepine
Long
9
short
The 72
ADR WORKBOOK SERIES - ENDOCRINOLOGY

Mainstay of treatment of nephrogenic diabetes insipidus is -


1. Desmopressin
2. Thiazide/amiloride diureties and salt restriction
3. Desmopressin and salt restriction
4. Vasopressin and salt restriction

Treatment of Nephrogenic diabetes insipidus:

DOC Hydrochlorothiazide

cnn.yy.am oq
Refractory NSAIS

Foot
no

X
y

H2O

reabs from f

PI
H2O

10
ADR WORKBOOK SERIES - ENDOCRINOLOGY
SIADH

syndrome of inappropriate ADH


PAIN
SIADH secretion is seen in all except -
1. Lung abscess H2o reabsorption
2. Interstitial Nephritis
3. Vinka alkaloids
4. Bronchial adenoma
fluid volume
ETIOLOGY

Neoplasm: Metabolic Vascular

Carcinomas Acute intermittent •Cerebrovascular


porphyrias occlusion,
•Lung Head trauma Bronchial asthma hemorrhage
•Duodenum Pneumothorax •Cavernous sinus
•Pancreas Infections Pneumonia, thrombosis
•Ovary bacterial or viral
•Bladder, • Pneumonia Positive-pressure
Ureter • Abscess (lung or brain) respiration
•Cavitation(aspergillosis)
• Tuberculosis, lung or
brain Drugs Neurologic
Other • Meningitis, bacterial or
Neoplasms viral Vasopressin or Guillain - barre-
• Encephalitis desmopressin syndrome
• Thymoma • AIDS Chlorpropamide Multiple sclerosis
• Mesothelioma Oxytocin, high Delirium tremens
•Bronchial dose Vincristine Amyotrophic
adenoma Cyclophosphamide lateral sclerosis
• Carcinoid Tricyclic Hydrocephalus
• antidepressants Psychosis
Gangliocytoma Monoamine Peripheral
•Ewing oxidase inhibitors neuropathy
sarcoma Serotonin
reuptake inhibitor
SIADH is associated with -
1. Small cell carcinoma lung
2. Adenocarcinoma lung
3. Squamous cell carcinoma lung
4. Mixed cell tumor lung
11
ADR WORKBOOK SERIES - ENDOCRINOLOGY

All are true regarding SIADH, except -


1. Increased level of ADH
2. Urine hypo osmolar
3. Hyponatremia
4. Adequate hydration status

Compensatory Mechanisms in SIADH: -


Normalize fluid levels

RAAS AND sympathetic

Atrial natriuretic out flow

IFluidvolumef
IBIpept.de
BP NORML
Renal Perfusion Distended Atria

JG Apparatus Renin Natriuresis

Nat H2o out


Angiotensinogen AT I
LACE
If Hyponatremia
ATTID
It 12 Eurolemic 4M
Aldosterone
I fluida
Not 6 H2o A
BPA
ADR WORKBOOK SERIES - ENDOCRINOLOGY
Hyponatremia may be seen in all of the following conditions except -
1. Mucoviscidosis
2. SIADH
3. Diabetes insipidus
4. Adrenal hyperplasia

Causes of Dilutional hyponatremia: - CHF

Cirrhosis Liver

CRE
SIADH - all are features except -
1. Decreased sodium, maintaining the concentrating ability of the urine osmolality
2. Normal sodium balance maintained indicating excess urinary sodium is due to
efficient sodium intake
3. Hypouricemia
4. Low blood pressure due to volume depletion

Clinical features of SIADH: -


1

t.IT
Altered Head
seizures Death
sensorium ache
Mat Normal BP

No sto fluid overload No Ascites


Nat
13 no edema
1

135 145
ADR WORKBOOK SERIES - ENDOCRINOLOGY osmolality
Osm kg
Sr Osm
osmolarity
Uriosm Ur Not a
Osm liter
Sr Not
Laboratory features of SIADH: -

Water loading test in SIADH: - t


101
consumption
Sr uric acid ked In SIADH
Dilution effect
1
Seen in SIADH-
1. Generalized edema
2. Ascites
3. Normal BP
4. Dry mucous membrane

SIADH true is all except -

1. Serum Na can be less than 135 meq/1


2. Urine sodium is normal or slightly low
3. Vaptans are new FDA approved drugs for its Rx
4. Water loading test can be used

Management of SIADH: -

Fluid restriction: - Ist line R

800mel 24h1

14

Correct Nat 0.5 meal 3


Mad
ADR WORKBOOK SERIES - ENDOCRINOLOGY

Vasopressor receptor antagonist (Vaptans): -

V2 R Antagonist V G V2 R antagonist Vi R
conivaptan antagonis
Tolvaptan oral
IV
Lixi Unmozavaptan
Role of Furosemide in SIADH: -
Relcovaptan

pulmonary edeme or else F C I


Which drug is not used in SIADH -
1. Fludrocortisone
2. Demeclocycline
3. Desmopressin
4. Hypertonic saline

True about SIADH is?


1. Euvolemic hyponatremia
2. Euvolemic hypernatremia
3. Hypervolemic hypernatremia
4. Hypervolemic hyponatremia

All are true about Diabetes insipidus except?


1. Low urine osmolality
2. Dilutional Hyponatremia
3. Water deprivation test is used for diagnosis
4. Polyuria

15
ADR WORKBOOK SERIES - ENDOCRINOLOGY

All are correct about SIADH except:


1. Normal KFT
2. Low uric acid 3 Polyuria 6
3. Relative hypernatremia
Not
4. Normal BP with gain of water polydipsia
1st and 3rd
SIADH secretion is seen in all except: sv.am A N
1. Lung abscess
2. Interstitial Nephritis
3. Vinka alkaloids
4. Bronchial adenoma
Urosm

DI PP DM
A 35-year-old man presents with vomitings and confusion. On examination Na+
120 m mol/L, K+ 4.2 m mol/L, Uric acid 2 mg/dl. Patient is not edematous. The
diagnosis is:
1. Cerebral toxoplasmosis with SIADH
2. Hepatic failure
3. Severe dehydration
II SIADI
4. Congestive heart failure

ADHI APHA

All are true regarding SIADH except: Srosm A Srosm


1. Increased level of ADH
2. Hyposmolar urine
Uriosm a
3. Hyponatremia
4. Adequate hydration status
Sv Not a

Pituitary Urnat IN
Which of the following is the most common type of pituitary adenoma?
1. Thyrotropinoma
2. Gonadotropinoma
Desmopressin VAPTANS
3. Prolactinoma
4. Corticotropinoma
MC secretory HCT
16

WEI KUI
ADR WORKBOOK SERIES - ENDOCRINOLOGY

Pituitary Tumors

Classification of Pituitary Adenomas

Adenoma Cell Origin Hormone Product Clinical Syndrome


Lacto trope PRL Hypogonadism, galactorrhea
Gonadotrope FSH, LH, subunits Silent or hypogonadism
Somatotrope GH Acromegaly/gigantism
Corticotrope ACTH Cushing’s disease
Mixed growth hormone GH.PRL Acromegaly, hypogonadism,
and prolactin cell galactorrhea
Mammosomato trope PRL,GH Hypogonadism, galactorrhea,
acromegaly
Thyrotrope TSH Thyrotoxicosis
Null cell None Pituitary failure
Oncocytoma None Pituitary failure

Prolactinomas Lactotroph cells

Middle age
MICOSMacmaammTAPH.CICIF
April
210mm 10mm
MASS
All are causes of hyperprolactinemia, except -
1. Bromocriptine
2. Phenothiazine
effect
3. Methyldopa
4. Metoclopramide

17
ADR WORKBOOK SERIES - ENDOCRINOLOGY

Etiology of Hyperprolactinemia

physiological pathologies

pregnancy t.IT
Lactation Tumors Systemic Drugs
Sleep REM CRF
Prolactinoma cirrhosis
stress
Cimetidine
Nipple 4 craniophoiryngionytHYPI.metac.io

promide
Phenothiazine Risperidone
A 30-year-old woman presented with secondary amenorrhoea for 3 years along with
galactorrhoea. The most likely cause of her symptoms would be -

1. Craniopharyngioma
2. Prolactinoma
3. Meningioma
4. Sub-arachnoid haemorrhage

18
ADR WORKBOOK SERIES - ENDOCRINOLOGY
Clinical features of prolactinoma: -

mass
Endocrine
effect
Bitemporal
Lactation Hemianopic
Libido

Infertility Infertility
Amenorrhea BI superio
quadrantanope

e
D

XA
19
ADR WORKBOOK SERIES - ENDOCRINOLOGY

Ramkali Bai, a 35 year old female presented with one year history of menstrual
irregularity and galactorrhoea. She also had off and on headache her
examination revealed bitemporal superior quadrantopia. Her fundus examination
showed primary optic atrophy. Which of the following is most likely diagnosis in
this case
1. Craniopharyngioma
2. Pituitary macroadenoma
3. Ophthalamic IcA Aneurysm
4. Chiasmal Glioma

CRANIOPHARYNGIOMA
Bimodal Age
Mass Effect

Young G
On Optic Chiasms On Pituitary

Ederly
 Inferior bitemporal Diabetes insipidusQ
hemianopiaQ Physiological and
 Optic atrophyQ mental radiationQ
 PapiIledemaQ
 Headache, vomitingQ

BT interior guardrafanopia

INVESTIGATIONS

Investigation of choice of hypeprolactinemia-


1. TRH estimation
2. LH estimation
3. Prolactin estimation
4. Estradiol estimation

20
ADR WORKBOOK SERIES - ENDOCRINOLOGY
Diagnosis of Hyperprolactinemia: -

Prolactin Normal Level: -


IC N 5 20
agld
i
2200
prolactin
100 200
41
one
MRI of Pituitary Gland: - SI
A Young woman with secondary amenorrhoea and galactorrhoea. MRI shows a
tumour of <10mm diameter in the pituitary fossa. Treatment is -
1. Hormonal therapy for withdrawal bleeding
2. Radiotherapy
3. Chemotherapy
4. Bromocriptine
5. Surgery

Medical management

MANAGEMENT OF PROLACTINOMA

Elevated Prolactin level

Exclude secondary causes of hyperplactinemia


MRI evidence for pituitary mass

Symptomatic Prolactinoma

Microadenoma Macroadenoma

Dopamine agonist Dopamine agoinst

Microadenoma
prey Bromo
21
criph Macroadenoma

Non prea
later golin
ADR WORKBOOK SERIES - ENDOCRINOLOGY

Dopamine Dopamine
agonist agonist

Serum PRL Repeat MRI


within 4 months

<20 20-50 >50(g/L)

No tumour Tumour
shrinkage or shrinkage and
Maintenance Reasses Dx. tumour growth or prolactin
Rx Increase dose persistent normalize
hyperprolactinemi
a
Consider Monitor PRL
surgery And repeat
MRI

Indications for surgery in case of prolactinoma: -


Dopamine Agonist

Hypopituitarism intolerance
Definition: -
partial complete loss of anteriorpituitary

functif
f LH FSH
1st GH
Hormone

22
ADR WORKBOOK SERIES - ENDOCRINOLOGY

Etiology: -

• Large pituitary tumors, or cysts, as well as hypothalamic


tumors (craniopharyngiomas, meningiomas, gliomas) can lead
to hypopituitarism.
• Pituitary adenomas are the most common cause of
panhypopituitarism.
• The mass compresses the gland, causing pressure, trauma, and
necrosis.
• Pituitary apoplexy is a syndrome associated with acute
hemorrhagic infarction of a preexisting pituitary adenoma, and
manifests as severe headache, nausea or vomiting, and
depression of consciousness.
• It is a medical and neurosurgical emergency.

Etiology: -

Inflammatory diseases can lead to hypopituitarism:

• Granulomatous diseases (sarcoidosis, tuberculosis [TB], syphilis),


eosinophilic granuloma, and autoimmune lymphocytic hypophysitis
(usually associated with other autoimmune diseases such as
Hashimoto thyroiditis and gastric atrophy).
• Trauma, radiation, surgery, infections, and hypoxia may also
damage both the pituitary and hypothalamus.
• Vascular diseases such as Sheehan postpartum necrosis
(initial sign being the inability to lactate) and infiltrative
diseases including hemochromatosis and amyloidosis may
induce this state as well.
• Stroke can also damage these cells. Stroke can cause central
diabetes insipidus due to damage of hypothalamus and/or
posterior pituitary.

23
ADR WORKBOOK SERIES - ENDOCRINOLOGY
Clinical Findings: -

Diagnosis: -

 Diagnosis. The first step in diagnosing pituitary insufficiency is to


measure GH, TSH, LH, and IGF-1.
 The most reliable stimulus for GH secretion is insulin-induced
hypoglycemia.
 After injecting 0.1 µ/kg of regular insulin, blood glucose declines to <40
mg/dL; in normal conditions that will stimulate GH levels to >10 mg/L and
exclude GH deficiency.
 Random GH and IGF levels are not sensitive enough to diagnose GH
deficiency.
 This is why a provocative test is used.
 Arginine infusion can also stimulate growth hormone release.
 Measure GH levels after infusing arginine.
 This is less dangerous because it does not lead to hypoglycemia.
 To diagnose ACTH deficiency, basal cortisol levels may be preserved (the
problem could be only in response to stress).
 Insulin tolerance test is diagnostic and involves giving 0.05-0.1 U/kg of
regular insulin and measuring serum cortisol; plasma cortisol should
increase to >19 mg/dL.
 Metyrapone tests for decreased ACTH production.
 Metyrapone blocks cortisol production, which should increase ACTH
levels.
 A failure of ACTH levels to rise after giving metyrapone would indicate
pituitary insufficiency.
 Cosyntropin (ACTH) stimulation may give abnormally low cortisol
output if pituitary insufficiency has led to adrenal atrophy.
 To diagnose gonadotropin deficiency in women, measure LH, FSH, and
estrogen.
 In males, gonadotropin deficiency can be detected by measuring LH,
FSH, and testosterone.
 To diagnose TSH deficiency, measure serum thyroxine (T4) and free
triiodothyronine (T3), which are low, with a normal to low TSH.

24
ADR WORKBOOK SERIES - ENDOCRINOLOGY
Management

1st line DOC


1st line steroids LH
Hypopit
Dexamethasone FSH

fteroids
A patient was prescribed bromocriptine for prolactinoma and responded to her
symptoms. What is it's mechanism of action?
1. D2 receptor partial agonist
2. Increases prolactin levels
3. Normalizes serum prolactin levels
4. D2 receptor antagonist

Which of the following is not seen in hypogonadotropic hypogonadism?


1. Decreased FSH, LH
2. Decreased testosterone
3. Decreased prolactin
4. Oligospermia

Which is the most common tumour of pituitary?


1. Non-functioning adenoma
2. Prolactinoma
3. ACTH producing adenoma
4. Oncocytoma

Which is the most common cell involved in non-functioning pituitary adenoma?


1. Gonadotropin producing cell
2. Prolactin producing cell
3. GH producing cell
4. TSH producing cell

For galactorrhea and amenorrhea syndromes, additional investigation apart from


serum prolactin?
1. TSH
2. LH
3. Urinary Ketosteroids
4. HCG

25
ADR WORKBOOK SERIES - ENDOCRINOLOGY
Disorders of Growth Hormone: -

IF GHIR
GH children AGH Adults
children
resistance
Gigantism Acromegaly Dialartism
Laron

• Acromegaly - Definition: -
Somatotroph cells GH

Regulation of GH

Liver
Control of growth hormone secretion. GHRH = growth hormone = releasing hormone;
IGF = insulin – like growth factor; SRIF = Somatotropin release – inhibiting factor

26
ADR WORKBOOK SERIES - ENDOCRINOLOGY It Malignan
Etiology: -

somatotroph Mammo Somatotroph Micro Maer

AGH April AGH Mostly Benign


Clinical Findings: - 3 5ᵗʰ decade

Its Abnormal CNS


a soft
ABony
Tissue sm Local
over growth

t.tt I
61
If
Skint Macro Hepatomegaly CUS

Thickness glossio LVH polyps


spleeno

typerhydrosis OSI akidney.HN


HOLM 40415
Thyroid
skin tags Arrhythmias colonoscopy

CHF

A tmeaboism Death
Local CNS

t.ie
Head ache
Dyslipidemia Mammes
Hyperglycemia
Diplopia
27
Galactorrhea visual defects
Infertility
y
Loss of libido
ADR WORKBOOK SERIES - ENDOCRINOLOGY
Broad

t.fi
Extremities fate
o
Joints

Broad hands frontalbossing


osteof
G H Cirm red
foot prognathism
fingers
Statollen Malocclusion Teeth
Clinical manifestations of Acromegaly:
Local Tumor Effects: Sleep Disturbances Visceral Manifestations
 Visual field defects  Sleep apnea  Macroglossia
 Cranial nerve  Narcolepsy  Hepatomegaly
palsy(diplopia) Skin Manifestations  Splenomegaly
 Headache  Hyperhidrosis  Thyroid enlargement
Somatic Manifestations  Skin tags Sexual Function
 Acral enlargement Colon Manifestations  Menstrual
 Thickening of soft  Polyps abnormalities
tissue of the  Galactorrhea
hands/feet Cardiovascular
(increased Manifestations  Decreased libido
ring/shoe size)  Left ventricular Carbohydrates
Musculoskeletal hypertrophy  Impaired glucose
Manifestations  Hypertension intolerance
 Prognathism  Congestive heart  Insulin resistance
 Malocclusion failure  Hyperinsulinemia
 Arthralgias  Diabetes mellitus
 Carpal tunnel  Lipids
syndrome
 Hypertriglyceridemia
 Frontal bossing

Hypoglycemia Glucagon
28 Thyroid H Glucose
CRH GH catecholamines
GIC
Steroids GH NH
ADR WORKBOOK SERIES - ENDOCRINOLOGY
Investigations: -

IGF 1 9 5 times
screening test
First line: -

N 90 370mg MI
Glc
Levels α severity of 1s
GH suppresiontes
Glucose tolerance test
Confirmatory test: -

GH levels
100g Glc 1009

of GH secretion Glucose load confirms


failure
Random GH Assay: - To value in diagnosis
secreted pulsatile
Due to pulsatality GH secretion

Radiologic studies: -

MRI pituitary pit Adenomy

Fruits
Isolated spade shaped
digit

29
ADR WORKBOOK SERIES - ENDOCRINOLOGY

Lateral view
X Ray foot
Heel pad thickness a

N 13k021m Myxedema
Acromegalyd
Management: -
D phenytoin
callus
Obesity
MM peripheral edema

iI
Bromocriptine
ide PEGVISOMANT
Refractory
Lanveotide GH R antagonist cases
sagerio IGF 1
AIE cholecystitis Recurrence t octreotic
Complications: -

a
DM
CHF
i Insulinoma a Glucagono
MQ
visual field X ii carcinoid syndrome
Cord compression
Which is NOT a side effect of GH administration?
IV VIPOMA secretory
1. Gynecomastia
2. Hypoglycemia diarrhea
3. Slipped capital femoral epiphysis
4. Pseudotumor cerebri

30
ADR WORKBOOK SERIES - ENDOCRINOLOGY
CUSHING'S SYNDROME

Adrenal Gland Is
t.ie
cushingsconnsAddiponspheo
to

f
asteroids Ald IA cortex

cateche
elftremities
Lemon on match stick

Physiology of Steroids: -

Hypothalamus CRH

Iggy
yy ff
steroids ACTHA

31
ADR WORKBOOK SERIES - ENDOCRINOLOGY

The most common cause of Cushing's syndrome is-


1. Pituitary adenoma
2. Adrenal adenoma
3. Ectopic ACTH
4. Adrenal carcinoma

Etiology of Cushing’s -

I ACTH
ACTH dependent
independent

it Adrenal
pituitary
Adenoma isit.ian
i adenoma
Adrenal
corticotroph pheo
Medullary Ca Thyroid
Pancreatic Bronchial

carcinoids

Fi icI.ecicaceitActit
P Adenoma out cell ca
indepen
P Adenoma Aldeno
Iatrogenic
Cushing’s triad is characterized by a widened pulse pressure bradycardia, and
A
irregular respirations (also known as Cheyne-Stokes respirations)
m
32

Cushing's dls pit Adenoma


Cushing's syndrome All etiologies
ADR WORKBOOK SERIES - ENDOCRINOLOGY

Cushing's syndrome is not a feature of -


1. Adrenal carcinoma
2. Oat cell carcinoma of lung
3. Medulloblastoma
4. Pituitary adenoma

SYMPTOMS AND SIGNS OF CUSHINGS SYNDROME:

Earliest Clinical features: -


weight Gain central obesity

Insulin resistance IDM


Features of Fat redistribution: -
Lipolysis AFFA Redistribution

t.tt
Supraclavicular Centrod
Buffalo RoundGe
obesity
hump swollen filling
face
Dermatological manifestations:

1 Hyperpig
1

Skin Atrophy Purple fungal


Easy
bruisability striae int
to
ACTH
5 Corneum
slefat T.ve sicolor
Loss of dependent
33 candida cushings
ADR WORKBOOK SERIES - ENDOCRINOLOGY

Easy bruisability

striae

Hyperpigmentation

Clinical Features of Cushing’s Syndrome A. Note central obesity and broad,


purple stretch marks (B close-up) C. Note thin and brittle skin elderly
patient with Cushing’s syndrome D. Hyperpigmentation of the knuckles in
a patient with ectopic adrenocortical-tropic hormone (ACTH) excess.

Menstrual irregularities

oligomenorrhea Amenourbed

Signs of adrenal androgen excess


In I cushings

Hirsutism virilisation

aed Libido
Acne 34

oily facial skin


ADR WORKBOOK SERIES - ENDOCRINOLOGY

True about cushing syndrome -


1. Red striae present
2. Increased adrenalin
3. Proximal muscle weakness
4. Oedema

Proximal muscle wasting and weakness (Proximal myopathy)


steroid proteolysis

Loss of muscle mass Distal


pugimal
Bone loss (osteoporosis)
t osteoclastic Activity

red pathological fracture


Features not seen in Cushing's Syndrome is-
1. Hypoglycemia
2. Hypertension
3. Frank psychosis
4. Hypokalemia

CNS Steroid ABP a sensitivity of


catecholamines
HTN

LUH

steroids ENOL a
ELOPIC ACTH

Hypo kt
35
Metabolic alkalosis
ADR WORKBOOK SERIES - ENDOCRINOLOGY
Glucose intolerance:

Hepatic Gluconeogenesis

Hyperglycemia
Hypertension and Cardiovascular risk:
8AM red
Thromboembolic Events
Red 11PM 12AM Led
Neuropsycholosical changes and cognition

steroids A1
Cushings psychosis Ge Emotional outburst
All are features of cushings disease except-
1. Central obesity
2. Episodic hypertension
3. Easy bruising
4. Glucose intolerance

A 23-year-old woman presents to the clinic complaining of months of weight


gain, fatigue, amenorrhea, and worsening acne. She cannot precisely identify
when her symptoms began, but she reports that without a change in her diet
she has noted a 12.3- kg weight gain over the past 6 months. She has been
amenorrheic for several months. On examination she is noted to have truncal
obesity with bilateral purplish striae across both flanks. Cushing's syndrome is
suspected. Which of the following tests should be used to make the diagnosis?
1. 24-hour urine free cortisol
2. Basal adrenocorticotropic hormone (ACTH)
3. Corticotropin-releasing hormone (CRH) level at 8 a.m.
4. Inferior petrosal venous sampling
36
ADR WORKBOOK SERIES - ENDOCRINOLOGY

Investigations: -

Loss of diurnal
Earliest biochemical change
variation

24hr urinary levels of


Iinea cortisol
overnight
Dx test
Increasedd

AC

ACTH dependent
ACTH independent
Adrenal pathology
Normal 90 MRI pituitary EA
I
CT MRI Abdomen NO further
High dose Dx 1 test Work U

ACTH
ACTAX Inferior petrosal
37

I sinus
t yAdenoma Ectopic ACTH
sampling
h
1 119 CTMRI Thorax
ADR WORKBOOK SERIES - ENDOCRINOLOGY
Which of the following would you do first to confirm the etiology of Abdomen
hypercortisolism?
1. ACTH level
2. Inferior petrosal sinus sampling
3. MRI of the pituitary
4. Low dose dexamethasone
5. High-dose dexamethasone suppression testing

Which of the following would you do first to confirm the etiology of hypercortisolism
in a person with an elevated ACTH level?
1. Inferiorpetrpsal sinus sampling
2. CT of the adrenals
3. High-dose dexamethasone suppression testing
4. Corticotropin-releasing hormone stimulation and petrosal

Which of the following would you do first to confirm the etiology of hypercortisolism
in a person with a decreased ACTH level?
1. MRI of the pituitary
2. CT of the adrenals
3. High-dose dexamethasone suppression testing
4. Corticotropin-releasing hormone stimulation anti petrosal sinus sampling

Which of the following would you do first to confirm the etiology of hypercortisolism
in a person whose ACTH level suppresses in response to high-dose dexamethasone?
1. Inferior petrosal sinus sampling
2. MRI of the pituitary
3. CT of the adrenals
4. CT scan of the chest
5. Corticotropin-releasing hormone stimulation and petrosal sinus sampling

38
ADR WORKBOOK SERIES - ENDOCRINOLOGY
50 years old, a chronic smoker, presents with history of hemoptysis. He was having
truncal obesity and hypertension. He had an elevated ACTH level which was not
suppressible with high dose dexamethasone. What would be the most probable
diagnosis-
1. Bilateral adrenal hyperplasia
2. Adrenal adenoma
3. Pituitary tumor
4. Ectopic ACTH producing lung cancer

 Summary of Diagnostic Testing for Hypercortisolism


 The first step is to confirm hypercortisolism with a 24-hour urine
cortisol.
 If the test is equivocal, do 1-mg overnight dexamethasone
suppression testing or late-night salivary cortisol.
 The most common wrong answer is random cortisol level.
 The second step is to confirm the source (location) of the
hypercortisolism with an ACTH level.
 ACTH low: CT of the adrenals
 ACTH high: MRI - Pituitary.
 MRI Normal – High dose Dx suppression
 No suppression: CT of the chest (look for ectopic focus)
 If the MRI of the pituitary is unrevealing, do petrosal sinus sampling
for ACTH possibly with CRH stimulation.
 This is the single most sensitive test of an ACTH-producing pituitary
adenoma.

Which of the following is the earliest manifestation of Cushing's syndrome -


1. Loss of diurnal variation
2. Increased ACTH
3. Increased plasma Cortisol
4. Increased urinary metabolites of Cortisol

39
ADR WORKBOOK SERIES - ENDOCRINOLOGY
The differentiating feature b/w Ectopic ACTH secretion and Cushing syndrome is -
1. Hypokalemic alkalosis
2. Clinical features of cushing syndrome
3. Hyperpigmentation
4. Hypertension

Treatment: -

Surgical: TOC
 Trans-sphenoidal pituitary surgery in Pit. adenoma
 Locate and remove ectopic ACTH source
 Adrenalectomy-U/L or B/L
 In B/L Adrenal adenoma with mass effect  B/L adrenelectomy
is done IV hydrocortisone drip and Post op Hydrocortisone
tablets
 Follow up ACTH incr. (due to static incr. in steroids) & there is
steroids hyperpigmentation of palmar and creases of sole
 This is called as NELSON SYNDROME
B Hydrocortisone
ACTH MSH Hyperpig
Medical Management

1T
Replacement Target
DOC Adrenolytic
Agents Glucocorticoids pituitary
Ketoconazole
to
Mittate
Adrenal enzyme Hydrocortisone cabergolin
I
IV Etomidate pasireotide
40 G MI
for small cell Ca Cisplatin Irinottecan
ADR WORKBOOK SERIES - ENDOCRINOLOGY

ACETATE

DEHYDROEPLANDRO-
TERONE SULFATE (DHEAS)
CHOLESTEROL

17β-HSD0
C20-22-lyase DEHYDROEPLANDRO
S-TERONE(DHEA)
C17-20-lyase 5-
17ɑ-Hydroxylase 17ɑ-HYDROXY-
ANDROSTE
PREGENOLONE PREGENEOLONE
3β- NEDIOL
3β-HS/isomerase* HSD/isomerse
3β – HSD /isomerase* 3β-
C17-28lyase HSD/iso
17ɑ-HYDROXY-
PROGESTERONE merase*
PROGESTERONE 17ɑ-hydroxylase ANDROST
------------------
ENDIONE 17β- TESTOS
21-Hydroxylase
 HSD TERON
21-hydroxylase E
DEOXYCORTISOL
11β-
SEX STERIODS
hydroxylase
DEOXYCORTICOSTERON
E
11β-hydroxylase CORTISOL

CORTICOSTERONE
GLUCOCORTICOIDS
18-hydroxylase

18-HYDROXYCORTICOSTERONE
*3β-HSD/isomearse =3β – hydroxysteroid
18-HSDX dehydrogenase5,4-isomearse

ALDOSTERONE 17β-HSD=17β hydroxysteroid dehydrogenase

18-HSD=18-hydroxysteroid dehyrdogenase
MINERAL CORTIOCIDS

41
ADR WORKBOOK SERIES - ENDOCRINOLOGY
The main indications for medical therapy of Cushing's syndrome include: -

CII to surgery

Recurrence after surgery

Occult ectopic ACTH

while waiting for surgery


while waiting for the effect of
pituitary radiation
A young hypertensive patient has serum K+ 2.8 meq/L and high aldosterone level
with decreased plasma renin activity. The likely cause is/are -
1. Renal artery stenosis
2. Ectopic ACTH syndrome
3. Diuretic therapy
4. Conn's syndrome

CONNS SYNDROME
Hyperaldosteronism

Most common cause of Conn's syndrome is -


1. Adrenal hyperplasia
2. Adrenal carcinoma
3. Adrenal adenoma
4. Pituitary ACTH hypersecretion

42
ADR WORKBOOK SERIES - ENDOCRINOLOGY

Physiology of Aldosterone function BP

t.ie
secretion CDSDCT ENac Action
2 G of
principle Channels Not GHzo
RAAS A cortex
cells kt

AT I Ht
Etiology of Mineralocorticoid Excess

21 Hyperald
Ie Hyperaldosteronism

MCC BK Adrenal hyperplasia CHF


cirrhosis of Liver
MCC conns Adrenal Adenema
HRS

NOT a feature of primary hyperaldosteronism is -


1. Pedal edema
2. Diastolic hypertension
3. Polyuria
4. Hypokalemia

43
ADR WORKBOOK SERIES - ENDOCRINOLOGY

PRIMARY ALDOSTERONISM

Increased secretion of mineralocorticoid

Increase retention of Increased Increased excretion


sodium and water excretion of of Hydrogen in
Potassium in exchanged of sodium
exchanged of
sodium

Diastolic HTN Arrhythmias metabolic

polyuria alkalosis
polyuria
polydipsia
No edemI
severe

Muscle weakness

44
ADR WORKBOOK SERIES - ENDOCRINOLOGY

ALDOSTERONE ESCAPE PHENOMENON: - Aldosterone NO 4H20


DCTGCD
fluid
COA VRA
900 AGFR ARP BPA
Laboratory anomalies: -
Aldosterone
I
Transient Anat
Renin 1 29
1kt ECG TI U weave
screening test ARE 424
ABG Met Alkalosis
Confirmatory tests for Primary Hyperaldosteronism: 7 1 HE

oral salt solution testT Aldosterone 11

Iv saline suppression test


11 11

Fludrocortisone suppression test

Captopril challenge test

What is the indication for genetic testing?


Adrenal venous
sampling
F Ho Hyperoldosteronism
To differentiat
Hyperaldosteronism young age
45
OIL from BIL
Adrenal CT MRI Large masses
Adrenal Adrenal tumors
d
Gland
ADR WORKBOOK SERIES - ENDOCRINOLOGY

Treatment: -
Toc surgical resection UIL
It BIL Adrenal tumors MM spironolactone

Eplerenone
D/D for CONNs is LIDDLE SYNDROME: -
Genetic dis

t.tt
Low renin DOC
AD Gain of Hypertension
HTN Amiloride
function
Triamteren
ENOL
A 32-year-old man presents to your clinic as a new patient to establish primary
care. He has a 2-year history of hypertension, which is managed with a calcium
channel blocker. He has no knowledge of the cause of his hypertension. He is
currently without complaints and only wants a refill on his medication. His
physical examination is unremarkable, but laboratory results show hypokalemia.
Which of the following statements regarding hyperaldosteronism is true?
1. The most common causes of secondary hyperaldosteronism are congestive
heart failure and cirrhosis with ascites
2. Treatment of primary adrenal hyperaldosteronism is spironolactone
3. Patients with primary adrenal hyperaldosteronism usually present with
hypertension, hypokalemia, and metabolic acidosis
4. Diagnosis of primary adrenal hyperaldosteronism is confirmed by elevations in
the levels of both rennin and aldosterone

Medicine made easy by Dr Rajesh Guble


46
10P 150
ADR WORKBOOK SERIES - ENDOCRINOLOGY

Testing for primary aldosteronism should be done for all hypertensive patients
except?
1. sustained hypertension above 150/100 mm Hg on 3 different days
2. hypertension resistant to three conventional antihypertensive drugs,
including a diuretic
3. controlled blood pressure requiring four or more antihypertensive drugs
4. Hyperkalemia, whether spontaneous or diuretic induced

Prior to testing for Hyperaldosteronism, patient ideally hold all medications except.

1. Diuretics
2. ACE inhibitors
3. NSAIDs
4. Verapamil

47
ADR WORKBOOK SERIES - ENDOCRINOLOGY
ADDISONS DISEASE
I
Adrenocortical 2types 10 Al

20 Al
Most common cause of adrenal insufficiency in India is-
1. Autoimmune
2. Surgery
3. Steroid withdrawal
4. Tuberculosis

Primary and secondary adrenocortical insufficiency

A Gland Ie pituitary Gland 29


I steroids Weak sex t
f steroids
MC weak sex.it MC Normal

FACTH MSH
ACTHA MSH A
Hyperpigmented skin Hypopigmented skin
Aetiology of primary adrenal cortical insufficiency

What is the MCC of Adrenal Insufficiency?

II India
West
TB_
Autoimmune 48

Adrenalitis
ADR WORKBOOK SERIES - ENDOCRINOLOGY

What are the bacterial infections which causes Addison's disease?

FB re meningitides

What are the fungal infections which causes Addison's disease?

Histoplasmosis

What are the Viral infections which causes Addison's disease?

CMV HIV
What is Adrenomyeloleucodystrophy?

X Linked dis and Long chain FFA


Neurological CIE
What are the infiltrative disorders causing Addison's disease?

Amyloidosis Sarcoidosis
Hemochromatosis
What are the malignancies causing Addison's disease?

Lymphoma Mets to A Gland


Leukemia
What are the causes of secondary adrenal cortical insufficiency?

Tumor Radiation pituitary


Trauma 49
Hypothalamus
surgery
ADR WORKBOOK SERIES - ENDOCRINOLOGY

Addison's disease is characterized by all except -


1. Hyperglycemia
2. Hypotension
3. Hyperkalemia
4. Hyponatremia

Adernal Insufficiency

Glucocorticoid Mineralocortic Adernal Other signs


deficiency oid deficiency androgen and symptoms
deficiency

fatigue Not Lack of


wt loss salt
craving
Energy
Dry a itchy

Gregg
HYPOTH
CIF of
skin
Hypoxat
Hypoglycemia
Loss of
fever Akt
Libido
Anemia Metabolic
Loss Axillary
Eosinophilia Acidosis

postural50Hypoxia PL
Lymphocytosis

OF
1 so
939 10
ADR WORKBOOK SERIES - ENDOCRINOLOGY

Pigmentation in Adrenal insufficiency:

Hyper AMSA t.se IMSA 2e Hypo

palm a sole creases


Areola Axilla Oral Mucosal
ALABASTER
LIN
fossa
Groin Cubital
Chronic adrenal insufficiency is characterized by all of the following except -
1. Excess pigmentation
2. Asthenia
3. Weight gain
4. Hypoglycemic episodes

Acute adrenal crisis & Waterhouse Friedrichsen syndrome:

20 to meningococcal infection N meningitides

children a new born R DOC IV

Hydrocortisone

Adrenal hemorrhage
Ivf res

Abdominal pain vascular collapse iv Dextrose


CI 15
A vomiting Coma
51
Ceftriaxone
OTN Death
ADR WORKBOOK SERIES - ENDOCRINOLOGY

Investigations
10C ACTH stimulation Co syntopin
test

Ie Aldosterone
act ofogue short synacthentes

20 Aldosterone N 250mg Sr Cortisol

CT abdomen –
Moth eaten
50onmo1 2500
Adrenal Gland
Normal Adrenal

Management of primary and secondary adrenal cortical insufficiency:

DOC Hydrocortisone Dexamethasone DOL

Replacement therapy for primary hypoadrenalism


Drug Dose
Glucocorticoid

00
Hydrocortisone 20-30 mg daily
e.g. 10 mg on waking, 5 mg at 12:00 h, 5 mg at
18:00 h
or
Prednisolone 7.5 mg daily 5 mg on waking, 2.5 mg at 18:00 h

rarely
Dexamethasone 0.75 mg daily 0.5 mg on waking, 0.25 mg at 18:00
h
Mineralocorticoid
Fludrocortisone 50-300 mcg daily

52
ADR WORKBOOK SERIES - ENDOCRINOLOGY

Glucocorticoids Duration of G Activity M Potency (Eq.


Action Activity Dose) in mg

000
1 HYDROCORTISON Short 1 1 20
E (CORTISOL)

2 CORTISONE Short Minimum 0.8 Least potent


(0.8) G. (25)
3 PREDNISONE Intermediate 4 0.5 5

4 PREDNISOLONE Intermediate 5 0.5 5


5 MEPREDNISONE Intermediate 5 0 4

6 METHYLPREDNIS Intermediate 5 0 4
OLONE
7 TRIAMCINOLONE Intermediate 5 0 4

8. FLUPREDNISOLO Intermediate 15 0 1.5


NE
9 PARAMETHASONE Long 10 0 2

10 BETAMETHASONE Long 25 0 Most potent


G. (0.6)

0
11 DEXAMETHASONE Long Maximum 0 0.75
(30)
MINERALOCORTICOIDS

12 DOCA 0 20

13 FLUDROCORTISO 10 250 Most


NE potent M
14 ALDOSTERONE 0.3 3000 Not used
(max.) clinically

All are true about the condition shown except


1. Hyperpigmentation
2. Hyperkalemia
3. Purplish striae
4. Metabolic alkalosis

s
cashing
B H

53
ADR WORKBOOK SERIES - ENDOCRINOLOGY

All are features of glucocorticoid deficiency except.


1. Weight loss
2. Fever
3. Hyperkalemia
4. Postural hypotension

Following are the common features of Cushing's syndrome except.


1. Truncal obesity
2. Osteoporosis
3. Distal myopathy
4. Glucose intolerance

Causes of diffuse hyperpigmentation include the following except:


1. Busulfan administration
2. Nelson's syndrome
3. Addison's disease
4. Hermansky-pudlak syndrome
HYPOPI

Investigation to be performed in a patient with hypertension and hypokalemia?


1. Renin aldosterone ratio
2. ACTH stimulation test
3. 24-hour Urinary Catecholamine's
4. Octreoscan
done

A 28-year-old lady has put on weight (10 kg over a period of 3 years) and has
oligomenorrhoea followed by amenorrhoea for 8 months. The blood pressure is
160/100 mm of Hg. Which of the following is the most appropriate investigation?
1. Serum electrolytes
2. Plasma cortisol
3. Plasma testosterone and ultrasound
4. T3, T4 and TSH lashings

54
ADR WORKBOOK SERIES - ENDOCRINOLOGY

A woman was admitted this morning in the medical intensive care unit for elective
cholecystectomy. Before surgery, her physical examination, including vital signs,
was normal. The procedure went well, and there were no noticeable complications.
However, 3 hours after returning to her room, she was noted to be unresponsive
and her blood pressure was barely palpable. She was intubated for respiratory
failure. Her blood pressure has been refractory to intravenous fluids and pressors.
You are consulted to help in the workup of suspected adrenal insufficiency.

Which of the following statements regarding adrenal insufficiency is true?

a. The most common cause of adrenal insufficiency in the United States is


tuberculosis

b. The critical test for the diagnosis of chronic adrenal insufficiency is the
cosyntropin test

c. Chronic secondary adrenalin sufficiency is treated with hydrocortisone and


fludrocortisone, whereas chronic primary adrenal insufficiency is treated with
hydrocortisone alone

d. In idiopathic or autoimmune adrenal insufficiency, CT of the abdomen shows


enlarged adrenal glands

55
ADR WORKBOOK SERIES - ENDOCRINOLOGY
PHEOCHROMOCYTOMA

Tumor of Adrenal medulla cells chromattin cells

E Neuroendocrine
Hormones E Dopamine

Paragangliomas: -

Extra Adrenal pheochromocytoma

predominantly NE
Paraganglioma
Chemodectomas
Location of Paragangliomas carotid
Location Percentage
Parasympathetic (non-secretory) head and neck 95 body
Catecholamine-secreting
Abdominal para-aortic 75
Urinary bladder 10
Thorax
Head and neck
10
3
Lyre sign
Pelvis 2

What is the MOST common extra-adrenal site of a pheochromocytoma?


1. Renal hilum
2. Mediastinum
3. Urinary bladder
4. Organ of zuckerkandl

All are clinical features of pheochromocytoma except-


1. Increased hematocrit
2. Orthostatic hypotension
3. Low cortisol level
4. Impaired glucose tolerance

56
ADR WORKBOOK SERIES - ENDOCRINOLOGY
CLINICAL FEATURES OF PHEOCHROMOCYTOMA: -
Ted demand
• On cardiovascular system: -
tiedsupply

f
Angina
fEPisodict Palpitations

sustained in absence of
H AChE
CAD
Arrhythmias
• Increase in metabolic rate 

ABMR wt loss
sweating
Temperature
• On Carbohydrate metabolism: -

Hyperglycemia
• Other manifestations of Pheochromocytoma: -

• Blunting of sympathetic reflexes 


Postural Hypoff

Negative
DID Addison's
feed back DAI
• Ectopic secretion of parathyroid hormone related proteins: -

57
ADR WORKBOOK SERIES - ENDOCRINOLOGY
First line Investigation in pheochromocytoma is -
1. CT scan
2. Urinary catecholamines
3. MIBG scan
4. Urinary calcium measurement

Dose of clonidine in suppression test done for pheochromocytoma is?


1. 0.3 mg
2. 10 mg
3. 100 mg
4. 200 mg

Investigations

1st line 24 hr Urinary Metabolites of


Catecholamines

VIA
confirmatory test
plasma metanephrine levels

411
4244m

photenotions Anxiety

clonidine Normal

60,68 DOTATATE PET SEEN 10


Imaging 58

Localizing the tumor


ADR WORKBOOK SERIES - ENDOCRINOLOGY
All of the following are increased in pheochromocytomas, except-
1. Vinyl mandelic acid (VMA)
2. Metanephrines
3. 5HIAA
4. Catecholamines

All of the following are features of Pheochromocytoma except-


1. Hypertensive paraoxysm
2. Headache
3. Orthostatic hypotension
4. Wheezing

Treatment TOC surgical resection Before Sx

Iii paragangliomy
oval phenoxy Intra OP Here
benzamine paroxysms Averbuch's CT
Phentolamine
Labeto protocol
in

IV Mitroprusside cyclophosphamide

IV
NTG vincristine
Ted sodic 6 cycles 21 day

sweating 59

Tachycardia CABMR
ADR WORKBOOK SERIES - ENDOCRINOLOGY
VMA is elevated in which of the following conditions?
1. Primary micronodular adrenal hyperplasia
2. Conn's Syndrome
3. Neuroblastoma
4. Tuberous Sclerosis

Which of the following is true about Pheochromocytoma:


1. Sestabimi scan is done before surgery
2. Mostly are malignant
3. Surgery is mainstay of treatment
4. Prior alpha blocker is given
5. Prior beta blocker is given

Drug for management of hypertension in Phaeochromocytoma?


1. Phenoxybenzamine
2. Phentolamine
3. Labetalol
4. Esmolol

In a case of Phaeochromocytoma, the diagnostic test best avoided is:


1. MRI scan
2. Urinary Metanephrines
3. MIBG scan
4. FNAC

60
ADR WORKBOOK SERIES - ENDOCRINOLOGY
Parathyroid disorders: -

PTH chief cells of PT Gland N0

factor 511014 16 A t p
Cat a osteoclastic activity a cat abs from renal tubules
Regulation of calcium

Cat in GIT indirect

1
Vit D calcitonin
PTH
7 46th Cath
Come

e p reaby
Renaltubules
Hyperparathyroidism: -

The commonest cause of primary hyperparathyroidism is -


1. Carcinoma parathyroid
2. Solitary adenoma of parathyroid
3. Chronic renal failure
4. Hyperplasia of the parathyroid

Etiology of hyperparathyroidism

Is other
11 7
Mcc Solitary PT CRF sq cell Ca
MCC Adenoma
61
Lung
Malabsorption
Multiple Adenoma Adeno.ca
Diffuse hyperplasia Vit D
kidney
Dttu yellas ey
carcinoma ADR WORKBOOK SERIES - ENDOCRINOLOGY

Drugs Bisphosphonates E Bone


resorptions
Alter Vit D
Phenytoin plictamycin
metabolism
Ketoconazole
Clinical features of Primary Hyper PTH: -
6 2 Bony
abnormalities
Due to Hypercalcemia: -
9 11mg dl

1 Tismgi
amgi.is ismg
Renal colic Bone pain systolic
Nephrolithiasis Depression arrest
eats oxalate 4 Psychosis
ca phosphate
21 parkinsonism
Cat induced ileus
constipation
Jan 1 AD
Detect PTH R
Abdominal pain
Gain of function
parietal cells 2
Ca
HCl PUDIS 62

EE i iiin
I 0lb do 1 ey
With abs kidney TH 96th urinary coil
ADR WORKBOOK SERIES - ENDOCRINOLOGY
Bone in hyperparathyroidism

I
Abnormal bony

remodelling
one I

Skull Mandible osteitis fibrosa


pin head stippling
Absence of L dura Mineralized
Cystic

Spine cod fish spine 6


Rugger Jersey spine
Osteitis fibrosa cystica:

Abnormal Bone
remodelling

63
ADR WORKBOOK SERIES - ENDOCRINOLOGY

• Spinal Deformities

cod fish
spine

• Rugger Jersey spine

• Pinhead stippling:

red resorption
Skull
DID
Multiplemyelomy

64
ADR WORKBOOK SERIES - ENDOCRINOLOGY

Absence of lamina dura:

Markers of hyperparathyroidism

IF
short interval DEXA
not of alk.pe Team
XP urinary cat to sestamibi SET
APTH Bone turn
scans I

si i ieCPtnVpSameAg

i.it iis
65

Normal osteopenia osteoporosis Normal osteo


I 1
porosis
ADR WORKBOOK SERIES - ENDOCRINOLOGY

In hyperparathyroidism, which of the following is not seen -


1. Normal alkaline phosphatase
2. Decreased phosphate in urine
3. Increased calcium
4. Hypophosphatemia

Alkaline Phosphatase: -
Active ph 9 other_
Placenta
Index of Liver a bone dis
Kidney
Intestines

Hepatic 6 Bony abnormalities Alk Poy Altered

III
a Normal

ptH Hypophosphatasia HypopTH


Rickets Multiple
osteomalacia
myeloma
Disproportional osteoporosis
Early Nymal
paget's
2 HyperPTH

66
ADR WORKBOOK SERIES - ENDOCRINOLOGY
Treatment of Hyperparathyroidism

TOC surgical resection

Indication asr.ca byImgldl it calciton


above the normalrange IIine DOI
of 24hr Urine INFT Bisphos nasal
Marked
cats excretion 490 furosemide phonates spray
2.5
3 T score
4 fed creatinine clearance
A 45-year-old man with a history of primary hyperparathyroidism comes to your
clinic for a follow-up visit. He was diagnosed 3 years ago after routine blood tests
revealed an elevation in calcium level. He has no complaints. Review of systems is
negative, and his physical examination is unremarkable. His family history is
negative for similar problems. His calcium level is 12.5 mg/dl. A DEXA scan shows a T
score of –2 at the hip. What is the most appropriate treatment regimen for this
patient?
1. Observation, with routine follow-up visits that include assessment of calcium
levels and DEXA scans
2. Start a bisphosphonate
3. Refer to an experienced surgeon for parathyroid surgery
4. Administer calcium, 1,000 to 1,500 mg/day, and vitamin D, 400 to 800 IU/day

67
ADR WORKBOOK SERIES - ENDOCRINOLOGY

A 54-year-old woman comes to your clinic for a routine visit. She has no active
complaints. Her medical history is positive only for mild asthma and arterial
hypertension. Her only medications are albuterol, which she administers with a
measured-dose inhaler as needed, and an angiotensin-converting enzyme inhibitor.
She smokes one pack of cigarettes a day. She has a strong family history of
osteoporosis. Her physical examination is unremarkable. You have a discussion with
her regarding her risk of osteoporosis, and you decide to obtain a dual-energy x-ray
absorptiometry (DEXA) scan for screening. The results show a T score of –2.6. Her
creatinine and albumin levels are normal, her liver function tests are normal except
for a slightly elevated alkaline phosphatase level, and her calcium level is 12 mg/dl.
What is the most appropriate step to take next in the treatment of this patient?
1. Start bisphosphonate, calcium, and vitamin D, and reassess in 6 months
2. Measure the parathyroid hormone (PTH) level with a two-site immunoradiometric
assay (IRMA, or so-called intact PTH) and assess 24-hour urinary calcium output
3. Order CT scans of the chest and abdomen to look for an occult malignancy
4. Start hormone replacement therapy with estrogens and progestins

HYPOPARATHYROIDISM

A 66-year-old woman presents to a walk-in clinic with muscle spasms. She


complains that for the past 2 days she has had muscle spasms in her hands, arms,
and legs. She has a medical history of cervical Hodgkin lymphoma, which was
treated with radiation. She does not take any medications or vitamins. On physical
examination, the Trousseau sign is positive. Her calcium level is 6.8 mg/dl; the
albumin level is normal.
On the basis of this patient's history, what is the most likely diagnosis, and what
should be the treatment?
1. Hypoparathyroidism secondary to radiation therapy; start PTH injections
2. Vitamin D deficiency secondary to poor intake and lack of sunlight; start
calcitriol
3. Vitamin D deficiency secondary to poor intake and lack of sunlight; start
cholecalciferol
4. Hypoparathyroidism secondary to radiation therapy; start calcium and calcitriol

68
ADR WORKBOOK SERIES - ENDOCRINOLOGY
HYPOPARATHYROIDISM

Causes: -

i
Autoimmunity Mg Aa Pigeorge

n
C A T C H 22
debt Hypocatt t
cardiac Palate ch 22
facies nce
Features: Reduced Ca+2 can be acute or chronic

Acute features - seen in thyroid Sx

I
Tetany 7mg
perioral a periangual paresthesias
Trosseou's 20mmHg of SBP carpal spasm

Chorster s Tap the facial N Angle of mandib


y
b
spasm of facial muscle
on that haft
69
DOC IV Ca Gluconate
Die Laryngospasm
surgically induced
g y
Hypo PTH IV Teri
paratide
ADR WORKBOOK SERIES - ENDOCRINOLOGY

For Chronic hypoCa+2:

Cranio tabes Skull Soft

Widening Wrist joint


Short stature
Wide opening Af
No teeth beyond 133months
Delayed dentition
GENU VALGUM GENU VARUM
“Gum” makes your “Rum makes your
knees stick together knees spread apar

KNOCK BOI

70
ADR WORKBOOK SERIES - ENDOCRINOLOGY
Investigations: -

PTA
2
16 ECG Long QI
P N 360 460msec

urinary cat
Treatment: -

oral calcium gluconate

Vit D supplementation

PSEUDO HYPOPTH PTH Normal

PTH R Resistant

PTH unable Action
A patient has hyperphosphatemia with short metacarpals and associated cataract.
The diagnosis is-
1. Pseudohypoparathyroidism
2. Hypophosphatasia
3. Hyperparathyroidism
4. Osteomalacia

71
ADR WORKBOOK SERIES - ENDOCRINOLOGY

Pseudohypoparathyroidism
 This is a hereditary disorder associated with signs and symptoms of
hypoparathyroidism despite an elevated PTH level (Hence called
pseudohypoparathyroidsm)
 This is produced as a result of deficient end organ response to PTH
 The most common form of Pseudo hypoparathyroidism (PHP- la) occurs in
association with Albrights Hereditary
Osteodystrophy and presents with distinctive skeletal and developmental defects

Laboratory Features Distinctive skeletal & Development


defects (Hereditary Albright's
• ↑PTH Osteodystrophy (AHO))

• Hypocalcemia features of • Short stature


• Round face
• Hyperphosphatemia
• Brachydactyly
hypoparathyroidism
(short 4^/5^ metacarpals and
• Normal/Decreased response of urinary metacarpals)
CAMP to PTH
• Heterotopic calcification
• Exostosis and Radius curves
•Impairment in olfaction & taste

72
ADR WORKBOOK SERIES - ENDOCRINOLOGY

Classification of Pseudo-hypoparathyroidism and Pseudo-pseudo-


hypoparathyroidsm (Harrison)
Various diverse forms of Pseudohypoparathyroidism (PHP) are recognized based
upon certain characheristics like urinary cyclic AMP response to exogenous PTH.
presence or absence of Albright's Hereditary Osteodystrophy (AHO) and assays
of concentration of the G5ɑ subunit of the adenylate cyclase enzyme.

Type Hypocalce Respon


GNASGsa
Serum AHO Resistance to
mia. se of PTH Subunit Hormones in
hyperphos Urinary Deficie Addition to
phatemia cAMP ncy PTH
to
PTH

Mc
PHP-
Ia
PHP-
Yes

Yes ↓
↓ ↑


Yes

No
OYes

No
Yes

No
Ib
PHP-ll Yes Normal ↑ No No No

O
PPHP No Normal Normal Yes Yes ±

Note
Pseudo Pseudo Hypoparathyroidism (PPHP) refers to a subset of patients who
carry the abnormal GNASI mutation / G5ɑ, subunit deficiency and have the
distinctive skeletal features of Herediatary Albright's osteodystrophy, but do not
show any evidence of endocrine or biochemical disturbance.
• The most common subtype of Pseudohypoparathyroidism (PHP-Ia) is typically
associated with reduced / decreased cAMP production in response to exogenous
PTH

Pseudohypoparathyroidism is associated with decreased formation of cyclic AMP


and not decreased cyclic CMP or Inositol Triphosphate
• Pseudohypoparathyroidism (Ia) results from an inherited 'loss of function'
mutation of the Gsa subunit
Gsɑ inactivating mutation (‘Loss of function’) Pseudohypoparathyroidis (Ia)

Taken from 'Principles & practice of Endocrinology and Metabolism' by Becher.


3rd/595

RR calcium Gluconate
73
ADR WORKBOOK SERIES - ENDOCRINOLOGY

Management of Pseudohypoparathyroidism

Oral Co2 Gluconate Vit D

K K KK
KI KK

Knuckle

KIK
f
THYROID DISORDERS

Steps in Synthesis of Thyroid hormone: -

1 Iodide trapping I

2 oxidation I Ie It Thyroid
Peroxidas
3 organification I It TG MIT Die

g up y mg
DIT MIT74 73
5 Deiodinization Ta nasty
ADR WORKBOOK SERIES - ENDOCRINOLOGY
The Lab investigation of patients shows decreased T3, decreased T4, & decreased
TSH. It cannot be -
1. Primary hypothyrodism
2. Pan – hypopituitarism
3. Secondary hypothyroidism
4. None of the above

Regulation of Thyroid hormone

737A F
IH Anterior pit TSH

7344 7 Gland
TSH
Primary and Secondary Thyroid disorders

74 TSH

Ie Hypothyroidism to

ze I

go Hyper in a

ze Hyper 75
ADR WORKBOOK SERIES - ENDOCRINOLOGY

generation TSH Assays-


1. 0.4 mlU/L
2. 0.04 mlU/L
00
Which of the following most closely represents the lowest detection limit for third

3. 0.004 mlU/L _Normal TSH 0.5 5mdull.it


4. 0.0004 mlU/L

TS ed 21 Hypo 1 Hyper
LL 1meulL

2nd LL 01mn12 severity


0.01 meull
Ird LL
Radioactive Iodine update
1123 I
32

High RAIU Low RAIU


• Graves disease • Subacute Thyroiditis
• Toxic nodule • Dequervain's thyroiditis
• Autonomous nodular Goitre • Viral thyroiditis
• Metastatic follicular thyroid • Lymphocytic thyroiditis
carcinoma
• HCG dependent hyperthyroidism of • Postpartum thyroiditis
trophoblastic • Exogenous hyperthyroidism
disease (Factitious hyperthyroidism)
• TSH secretin pituitary adenoma
• Recent iodine load (e.g.,
contrast dye)

Free T3, Free T4: -


73 6 To not bound to TBI
TBG 7374 Bound found
Hashimoto's thyroiditis, all are TRUE, EXCEPT-
1. Neutrophilic infiltration
2. Maximum incidence in child
3. Painless in Thyroid
4. None of the above

76
ADR WORKBOOK SERIES - ENDOCRINOLOGY

Hashimoto’s Thyroiditis

Autoimmune dis

Middle age It can occur in children

Mcc Ie Hypothyroidism
major cause of
Mcc Globally Ist non endemic Goiter

Lymphocytic infiltration Gland

Ab's Anti Destroy follicular cells


TPI I
some surviving follicular cell 7374
pink colloid HURTHLECel
Diffuse A Thyroid Gland
painless swelling
The Lab investigation of a patient shows decreased T4, & increased TSH. Which
of the following is the most likely diagnosis -
1. Grave's disease
2. Hashimoto's disease
3. Pituitary failure
4. Hypothalamic failure

77
ADR WORKBOOK SERIES - ENDOCRINOLOGY

Aetiology of Hypothyroidism
Suspected Hypothyroidism

Obtain TSH and Free T4

TSH High
TSH normal or
Free T4 Low
low free T4 low

Primary Hypothyroidism Secondary Hypothyroidism

Autoimmune hypothyroidism
Hashimot’s atrophic
Hypothalmic Pituitary
Iatrogenic:
131
treament, subtotal or total
thyroidectomy External irradiation Due to
Due to defective
of neck for lymphoma defective TSH
TRH Secretion by
Secretion by
Drugs: hypothalamus
pituitary
Iodine excess (iodine contract
media and amiodarone) lithium,
antithyroid drugs, paminosalicylic
acid interferon ɑ and other  Tumour  Tumours
cytokines, amniogluthimide  Trauma  Pituitary
sunitinib  Infiltrative surgery
disorder  Infiltrative
Congenital hypothyroidism:  'Idiopathic disorder
Absent or ectopic thyroid  Sheehans
dyshormonogenesis, TSH-R syndrome
mutation iodine deficiency  Trauma

Infiltrative disorders:
Amyloidosis, Sarcoidosis,
hemochromatosis, scleroderma,
cystinosis, Riedel’s thyroiditis
Overexpression of type 3 iodinase

78
ADR WORKBOOK SERIES - ENDOCRINOLOGY

Most common cause of hyperthyroidism- TDM


1. Thyroid hyperplasia
2. Thyroid adenoma HLADR
3. Thyroid carcinoma HLADR
4. Grave disease
DRG

MCC Grave's disease Autoimmune RA


HLA DRG
Ab's Thyroid Ig's
Etiology of hyperthyroidism

Hyperthyroidism

With a normal or high radioiodine uptake Hyperthyroidism with a near absent


radioiodine uptake
Autoimmune disease
• Graves disease Thyroiditis
• Hashitoxicosis • Subacute granulomatous (de quervains
thyroiditis)
Autonomous thyroid tissue • Painless thyroiditis (silent thyroiditis,
• Toxic adenoma lymphocytic)
• Toxic multinodular goitre • Amiodarone
• Radiation thyroiditis
TSH mediated hvperthyrroidism • Palpation thyroiditis
• TSH producing pituitary adenoma
• Non neoplastic TSH mediated Exogenous thyroid hormone intake
hyperthyroidism • Excessive replacement therapy
• Intentional suppressive therapy
Human chorionic gonadotropin • Factitious hyperthyroid
mediated hyperthyroidism
• Hyperemesis gravidarum Ectopic hyperthyroidism
• Trophoblastic disease

79
ADR WORKBOOK SERIES - ENDOCRINOLOGY

Causes of Thyroiditis

Acute Subactue Chronic

Bacterial infection Viral (or granulomatous) Autoimmunity:


• Fungal infection thyroiditis Hashimoto 's
• Radiation thyroiditis De Quervain ’s thyroiditis thyroiditis.
after • Silent thyroiditis Focal thyroiditis,
1311 treatment (including postpartum Atrophic thyroiditis
• Amiodarone thyroiditis) • Riedel’s thyroiditis
(may be subacute or • Mycobacterial infection • Parasitic thyroiditis:
chronic) • Drug induced • Traumatic: after
(interferon, amiodarone) palpation

Granulomatous thyroiditis

DeQuervain's Thyroiditis is characterized by a depressed radioactive iodine uptake'.

DeOuervain's Thyroiditis, Granulomatous Thyroiditis or Subacute Thyroiditis or Giant


cell Thyroiditis:
Origin / • It is viral in origin.Q Symptoms usually follow an upper respiratory
Etiology tract infection.
Clinical • Pain over thyroid Q or pain referred to lower jaw, car, occiput
presentation (stretching of thyroid capsule)
• Fever Q, malaise, asthenia
• Firm, irregular enlargement of one or both thyroid lobes

Characteristic • High Erythrocyte sedimentation rate (ESR)


Laboratory • Depressed RAIU i.e. decreased radio iodine uptake test
Finding • Early: patients may be mildly thyrotoxic due to leakage of hormone

8
from gland: ↑T3 & T4 & ↓TSH
• Later: patients may be hypothyroid due to depletion of hormone
from gland: T3 & T4 & ↑TSH
Histology • Multinucleated Giant cells Q are seen - Robbins
Prognosis • Disorder may smoulder for months but eventually subsides with
return of normal function.

80
ADR WORKBOOK SERIES - ENDOCRINOLOGY

The occurrence of hyperthyroidism following administration of supplemental iodine


to subjects with endemic iodine deficiency goiter is known as -
1. Jod-Basedow effect
2. Wolff-Chaikoff effect
3. Thyrotoxicosis factitia
4. De Quervain's thyroiditis

Wolf Chaikoff effect-


1. Excess iodine intake causes hypothyroidism
2. Iodine induced hyperthyroidism
3. Thyrotoxicosis due to excessive amount of iodine ingestion
4. Drug induced hyperthyroidism

Jod Basedow's disease & Wolf Chaikoff effect

Endemic Is Goiter

tfyperthyroidismt ENNNIHfhyroidi.sn
Jod Basedow's offo
Wolff - Chaikoff effect: -

Transient of organitication
Which of the following can alter thyroid function?
1. Quinidine
2. Flecainide
3. Amiadorone
4. Disopyramide

81
ADR WORKBOOK SERIES - ENDOCRINOLOGY

Amiodarone associated hyperthyroidism:


Two types of Amiadarone induces thyrotoxicosis (AIT) are seen -
AIT-Type I AIT-Type II
Associated with an underlying Associated with individuals with no
thyroid abnormality intrinsic thyroid abnormality
- preclinical gravis
- multinodular goitre
Thyroid hormone synthesis becomes Thyrotoxicosis results from drug
excessive as a result of increased induced lysosomal activation leading to
iodine exposure – Jod Basedow destruction thyroiditis.
phenomenon This can resolve spontaneously or can
occasionally lead to hypothyroidism
Increased vascularity is seen on Decreased vascularity is seen on color
color-flow dopier scan flow dopier scan
IL6 levels arc marginally raised. IL6, levels are markedly raised.

Clinical features of hypo and hyperthyroidism

physio Hyper Hypo

BMR a Heat intolerance cold intolerance

wt loss alt Gain

ned appetite fed appetite


profuse sweating Dry skin

CHI Glycogenolysis
Hyperglycemia
Gluconeogenesis
TG Chol
FFA 82 cholesterol
Lipid Lipolysis Atherosclerosis
TG CAD
ADR WORKBOOK SERIES - ENDOCRINOLOGY

Muscle fatigue
proximal muscle
proteolysis
myopathy

CVS Tachycardia Bradycardia


IHR
Foc Palpitations sBp
MC AF
exitability DBP
Irregular
VOL D HTN
irregular
SBP IDBP
SBP Ana Ie AV block

Irregl KRG DBP P'EFFUSI


Irregt p.pt
put
High out Heart
failure
Murmur MidsystolicM Means lermann

scratch
constipation
GI Gotility
Diarrhea

83
102962
Reenters Respiratory
Resist Tachyphed
failure
TRF
ADR WORKBOOK SERIES - ENDOCRINOLOGY
Clinical features of hypo and hyperthyroidism TIRF 102
202N to

t Extreme nervousness Dull 6 Lethargic


CNS
Anxiety neurosis Delayed relaxation

Aaa DTRS Of DTRs Hung up


reflex
Fine Tremors
Mostreliable sign

tied steel pan sleep


sleep
yellow skin
Pretibial myxedema
Skin
shin of
β carotenemI
Waxy skin
Tibia B carotene tvit A

Muller's muscle Eye Normal


Lid retraction Jaundice Icterus
Graves ophthalmopathy
Eyes
ophthalmopathy
GAGS
PV0P10 Infiltrative
smoking
Cytokines
secreted by fibroblasts
Earliest sign Liglag stare sign
84

Retrosternal Goiter pemberton sign Raising arms


facial congestion
ADR WORKBOOK SERIES - ENDOCRINOLOGY

All are seen in myxedema coma except-


1. Hypothermia
2. Tachycardia
3. Hypotension
4. Hyponatremia

Myxoedema coma
Abrupt in Thyroid hormone a

severe form of Thyroid I


Myxoedema coma

 Myxedema coma represents a state of severe hypothyroidism complicated


by hallmark neurological symptoms of reduced level of consciousness
(sometimes associated with seizures) and hypothermia.
 It is typically seen in in Elderly patients with history of long-standing
untreated severe hypothyroidism and presents with a constellation of
clinical manifestations
Clinical Manifestations
Reduced level of consciousness, sometimes associated with seizures
• Hypothermia (Hypothermia can reach 23°C (74°F))
• Hypoglycemia
• Hyponatremia
• Hypoventilation leading to hypoxia and hypercapnia
• Features of Hypothyroidism (or the patient may be previously undiagnosed)
 Myxedema coma has a 20-40% mortality rate, despite intensive treatment.
Since it has a high level of mortality this condition must be treated as a
Medical Emergency and aggressive treatment in Intensive Care Setting is
almost always required (Outcomes are independent of the T4 and TSH
levels)
Precipitating Factors
• Inadvertent administration of drugs such as hypnotics; narcotics, sedatives,
and/or antidepressants

0
• Sepsis
• Exposure to cold
• Hypoglycemia
• Other stressful events such as pneumonia, congestive heart failure, myocardial
infarction, gastrointestinal bleeding, or cerebrovascular accidents
Investigations
85
ADR WORKBOOK SERIES - ENDOCRINOLOGY
• Check for reduced T4 and Increased TSH levels
• Check for decreased Glucose and Hyponatremia
• Check for ACTH and Cortisol for evidence of adrenal insufficiency
Treatment
Aggressive treatment in Intensive Care Setting is required
• Ventilatory support with regular blood gas analysis is usually needed during the
first 48 h.
• Supportive therapy should be provided to correct any associated metabolic
disturbances.

IV glucose may be needed if there is severe hypoglycemia


Hypertonic saline may be needed if there is severe hyponatremia

T4/T3 Therapy

tube o
- Levothyroxine can initially be administered intravenously or via nasogastric

- An alternative is to give liothyroninc <T3) intravenously or via nasogastric tube


Liothyroninc (T3) treatment has been advocated because T4  T3 conversion

8
is impaired in myxedemacoma. However, excess liothyroninc has the potential
to provoke arrhythmias
- Another option is to combine levothyroxine and liothyronine

• External warming is indicated only if the temperature is <30°C. as it can result


in cardiovascular collapse Space blankets should be used to prevent further

0
heat loss.
• Parenteral hydrocortisone (50 mg every 6 h) should be administered, because
there is impaired adrenal reserve in profound hypothyroidism.
• Any precipitating factors should be treated, including the early use of broad-
spectrum antibiotics, pending the exclusion of infection.
• The metabolism of most medications is impaired, and sedatives should be
avoided if possible or used in reduced doses.
• Monitor for arrhythmia
• Note: Liothyronine (T3) has the potential to provoke arrhythmias
• Medication blood levels should be monitored, when available, to guide dosage.

86
ADR WORKBOOK SERIES - ENDOCRINOLOGY

Thyroid storm

 Thyroid crisis or storm -


 It is a rare disorder now days.
 It is extreme form of thyrotoxicosis which is precipitated by
stressful illness, thyroid surgery or Radioiodine administration.
 It presents with symptoms of thyrotoxicosis which are -
 Severe tachycardia, Arryhthmias
 Marked delirium
 Vomiting
 Diarrhoea
 Dehydration
 High fever
 High output heart failure - Cause of death
 Mortality rate is very high.

Rx- of thyroid storm:

 Stop IVF/Blood transfusion


 Propylthiouracil by NG tube - DOC  prevents conversion of T4
T3
 Ice packs
 Sodium bicarbonate
 IV Propranolol


o
For Toxic nodular goiter (preoperative)  Lugols I2 is given 
Regression of gland size, Decr. The production of
T4  Chance of thyroid storm is less.
 Lugols I2 acts by wolff chaikoff effect

87
ADR WORKBOOK SERIES - ENDOCRINOLOGY
A pregnancy woman is diagnosed to suffering from graves' disease. The most
appropriate therapy for her would be
1. Radioiodine therapy
2. Total thyroidectomy
3. Carbimazole parenteral
4. Propylthiouracil oral

Treatment of Hyperthyroidism: -

propranolol
Thy oxidase
resection
inhibitors
IHR DOC Methimazole
Destroys overact
Tremors
AE Agranulocytosis
thyroid tissue
Peripheral R
pregnancy avoided pregnancy

conversion
1st Tui PTO GeLactation
Dose 60 80mg
Teratogenic Aplasiacutis conceive
orally OD
2nd a 3rd Methimazole 4 months after

Abnormal
for upto Gmonth
spermato300
Intolerant to Drugs
after
1131 2months
pregnancy
FT4
Graves ophthd
88

Ale red HypoI


Choice Total resection 1 lobe
A 1
sub other is
ADR WORKBOOK SERIES - ENDOCRINOLOGY
49 of Thyroidtissue

A 40 yrs old female who is known case of ischemic heart disease (IHD) is diagnosed
having hypothyroidism. Which of the following would be most appropriate line of
management for her
1. Start levothyroxine at low dose
2. Do not start levothyroxine
3. Use levothyroxine
4. Thyroid extract is a better option

Treatment of Hypothyroidism: -

DOC Levothyroxine 16Mg kg


TA
check I
IHDIS

Yes No

fulldose
start x̅ low dose

Monitor 3 months

89
ADR WORKBOOK SERIES - ENDOCRINOLOGY

Measure TSH

Elevated Normal

Measure unbound T4 Pituitary disease suspected?

74
Normal Low No Yes

I
Mild hypothyroidism Primary Hypothryoidism No further Measure Unbound T4
tests

TPOAb+ TPOAb- low Normal


TPOAb+ or b
TPOAb-
symptomaic symptoms No further
Rule out other
Autoimmune tests
causes of

0
hypothryoidism
hypothryoidism
Rule out drug effects, sick
C Consider T4 Annula euthryoid syndrome,then
treatment follow-up T4 Treatment evalutate anterior pituitary
function

Evaluation of hypothyroidism

SUBCLINICAL HYPOTHYROIDISM TSH 6 74 N

Asymptomatil NO

symptomactive
LI
90
ADR WORKBOOK SERIES - ENDOCRINOLOGY

MEN Syndromes: -
Multiple endocrine neoplasia
All of the following are features of MEN 2a, except –
a) Pituitary tumor
b) Pheochromocytoma
c) Medullary carcinoma thyroid
d) Amyloidosis Magenmanfroboese
syndrome
Gene MEN 1 II RET 10 10 RET
MULTIPLE ENDOCRINE NEOPLASIA (MEN) SYNDROMES
MEN I (Wermer MEN 2A (Sipple syndrome) MEN 2B
syndrome) 13
P pituitary teem Thyriod medullary
g Ca Thyroid
P PTH
Mc
p pancreatic Adrenal Pheoch A
2nd
PTH Adenoma
Mc non secretory PPoms
Hyperplasia
MC secretory Gastrinoma

EE EE
A fasting GIC
c pep Insulinomy
Hirshprung's Mucosal GI
Insulin
carcinoid neuromas
Extra endocrine foregut Amyloidosis
Angiofibroma ganganygg
Collagenoma Martanoid
91
habitus
ADR WORKBOOK SERIES - ENDOCRINOLOGY
MEN - 4

Reproductiveorgan.am
Testicular tumors Pit
cervical tumors PTH

Adrenal Renal tumors

Gene CDKNIB Ch 12
Which of the following endocrine tumors is most commonly seen in MEN I?

1. Insulinoma
2. Gastrinoma
3. Glucagonoma
4. Somatostatinoma

1.
0
Werners syndrome is asscociated with?

MEN 1
2. MEN 2a
3.
4.
MEN2b
NONE
progeria

Type I MEN involves all, except –

1. Pancreas
2. Adrenal
3. Pituitary
4. Parathyroid

92
ADR WORKBOOK SERIES - ENDOCRINOLOGY

A patient presents with intermittent headache. On examination there is


hypertension and a thyroid nodule. Which of the following steps is to be taken
next-

1. Urine HIAA levels


2. Urine VMA and aspiration of the thyroid nodule
3. Ultrasound abdomen
4. Echocardiography

A young patient presented with HTN and VMA 14 mg/24, the causes is/are -

1. Medullary carcinoma thyroid


2. Von Hippel Lindau syndrome
3. Grave’s disease
4. Pseudohypoparathyroidism Men 2d
5. Sturge weber syndrome
2b

VRHNE
Autoimmune polyglandular syndromes

APS 2 APS 3
APS 1

At Adrenal Definitive Autoimmune


HYPOPTH
Thyroid
At Adrenal

Chroniccandidiasis
Athyroiddf 93
candidiasis

TIDAL At HYPOPIA
2
At Adrenal
ADR WORKBOOK SERIES - ENDOCRINOLOGY
DIABETES

Classification of DM

IF GDM
TDM TDM er

420485 30415

0
Which of the following endocrine does not lead to DM?

1. Pheochromocytoma
2. Somatostatinoma
3. Hypothyroidism
4. Acromegaly

Acromegaly

Hyperthyroidism

Somatostatinoma

94
ADR WORKBOOK SERIES - ENDOCRINOLOGY

TABLE - Etiologic Classification of Diabetes Mellitus

I. Type 1 diabetes (immune-mediated beta cell destruction, usually leading to absolute


insulin deficiency)

II. Type 2 diabetes (may range from predominantly insulin resistance with relative insulin
deficiency to a predominantly insulin secretory defect with insulin resistance)

III. Specific types of diabetes

A. Genetic defects of beta cell development or function characterized by mutations in:

O
1. Hepatocyte nuclear transcription factor (HNF) 4«(MODY 1)
2. Glucokinase (MODY 2)
3. HNF-lɑ(MODY 3)
4. Insulin promoter factor-1. HNF-lβ. NeuroDl. and others leading to other forms of
MODY
5. Insulin, subunits of ATP-sensitive potassium channel leading to permanent neonatal
diabetes
6. Mitochondrial DNA
LADAD
7. Other pancreatic islet regulators/proteins such as KLF11, PAX4, BLK,
Type 1.5
GATA4,GATA6,SLC2A2(GLUT2,RFX6,GLIS3
B. Transient neonatal diabetes DI
Type 3
C. Diseases of the exocrine pancreas—pancreatitis, pancreatectomy, neoplasia, cystic
fibrosis, hemochromatosis, fibrocalculous pancreatopathy, mutations in carboxyl
ester lipase

D. Genetic defects in insulin action, including type A insulin resistance. Leprechaunism.


Rabson-Mendenhall syndrome. Lipodystrophy syndromes

E. Endocrinopathies—acromegaly. Cushing’s syndrome, glucagoooma.


pheochromocytoma. hyperthyroidism, somatostatinoma. aldosteronoma

0
F. Drug- or chemical-induced—glucocorticoids, vacor(a rodenticide),pentamidine,

or
0
nicotinic acid, diazoxide. (β - adrenergic agonists, thiazides,
calcineurin and mTOR inhibitors, hydantoins. asparaginase, ɑ-interferon. protease
inhibitors, antipsychotics (atypicals and others), epinephrine

G. Infections—congenital rubella, cytomegalovirus, coxsackievirus

H. Uncommon forms of immune-mediated diabetes—"stiff person"


syndrome, anti-insulin receptor antibodies

Wolfram syndrome DIDMOAD


I. Other genetic syndromes sometimes associated with diabetes—
95
Wolfram's syndrome. Down's syndrome. Klmefelter's syndrome. Turner's syndrome.
Friedreich's ataxia. Huntington's chorea. Laurence-Moon-Biedl syndrome, myotonic
Infection Rubella CMV Coxsackie virus
dystrophy, porphyria. Prader-Willi syndrome

IV. Gestational diabetes mellitus (GDM)


ADR WORKBOOK SERIES - ENDOCRINOLOGY

All of the following statements are true regarding type 1 diabetes mellitus except.

1. Family history is present in 90% cases


2. Dependent on insulin to prevent ketoacidosis
3. Time of onset is usually abrupt
4. Autoimmune destruction of beta cells occur
5. Mostly occurs in children

Type 1 Diabetes Mellitus - Pathogenesis

Etiology HLADR DRG DQ


i Autoimmune destruction B cells Anti GAD

ICE

i fulminant DM 8 12 laleeks after viral infection

Immune system
Mumps Rubella Cox 1
ay
hogis Micro
Iii Bronze DM Hemochromatosis
Hyperpig skin

Immune mediated 1 T Lymphocytes

801 B cells TDM


Which type diabetes is HLA associated -

1. Type I diabetes
2. Type II diabetes
3. Malnutrition related type disease
4. Pregnancy related type diabetes

96
ADR WORKBOOK SERIES - ENDOCRINOLOGY
T2DM Pathogenesis: -

Risk factor
Insulin resistance Resistin
Centralobesity
Liver
6101 4 Insulin sedentary is

Y GIC vit Db
Smoking
TYPEI VERSUS TYPE2 DIABETES MELLITUS

Type I Diabetes Mellitus Type 2 Diabetes Mellitus

Age of onset
20415 30485
Weight
BMI normal obese
Genetics
6 101 1st order 70 80
Association with
HLA HLA DRS DRG
Pathogenesis
destruction Insulin resistance
B cell
Early rela Laterto
Islet cells
bed
Insulin level
feed a tied

Classic symptoms of
Polyuria, Polydipsia, Rare poorwound
Polyphagia
49 weakness AFever

Acute complications
DKA TI T HONK 1ST
Insulin necessary in
treatment
Insulin OHA 1st

97
Insulin
ADR WORKBOOK SERIES - ENDOCRINOLOGY

In a patient with T2DM which of the following condition is seen -

1. Ketosis commonly occurs on stopping treatment.


2. Hypertriglyceridemia never occurs
3. Pancreatic beta cells stop producing insulin
4. There are increased levels of insulin in blood

Dyslipidaemia is common in Type 2 diabetes mellitus.


Low dose statin
ATG MI 10mg Atorvastatin
Chol
HDL
LDL
VLDL
A 29-years-old male taking oral hypoglycemic drug never had ketonuria in his life.
His BMI is 20.5. His grandfather had diabetes and his father who is only son of his
grandfather too had the disease. Which type of DM this person will be most likely-

1. Pancreatic
2. MODY
3. Type I
4. Type II

Maturity onset diabetes of the young (MODY)-Rare form

t.tt
Pathogenesis
Normal BMI strong
Age
Never familyhistory
20 30415
2 Generations
on
yqn 98
ADR WORKBOOK SERIES - ENDOCRINOLOGY

Gene mutations MC HNF 12

DOC Love dose sulphonylurels

The genetic mutation seen in the most common type of maturity onset diabetes
of young (MODY) is-

1. Hepatocyte nuclear factor-4


2. Hepatocyte nuclear factor-1
3. Glucokinase
4. Insulin promotor factor-1

Type Genetic Optical treatment


defect
1 Hepatocyte
nuclear
factor-4
Low dose SU Glimeperide
alpha

2 Glucokinase
gene
Diet

3 Hepatocyte
nuclear so
factor-1-
alpha

99
ADR WORKBOOK SERIES - ENDOCRINOLOGY
4 Insulin
promoter
factor 1
So
5 Hepatocytc
nuclear
factor-l-
beta Insulin

6 Neurogenic
differentiati
on
factor-1 Insulin

MODY false statement is -

1. Age around 25 years


2. Impaired secretion of insulin
3. Responds to sulfonylureas
4. Insulin dependent

Type 1.5 DM (LADA):


Rare Latent Autoimmune Diabetes

ty.ly duIy
cell Insulint Anti
GAI R DOC
B
destruction
ICE Insulin
Diagnostic criteria for evaluation of standard Glucose Tolerance test according to
Diabetes expert committee.

100
ADR WORKBOOK SERIES - ENDOCRINOLOGY

Normal Glucose Impaired Glucose Diabetes


mellitus Targe
Fasting plasma
glucose (mg/dl)
4100 101 125 126 4130
Two hours after
glucose ingestion
(75g)
2140 141 199 7200 2180

HbA1C
45.6 5 7 64 765 27

A 42-year-old male has strong positive Benedict’s test, random blood sugar is > 163
mg%, fasting blood sugar is > 200 mg% Next line of investigation is-

1. Urine glucose charting 5 hourly


2. Oral GTT
3. Repeat benedict’s test
4. 24 hr urine sugar estimation

759
So any patient with plasma glucose value greater than 140 mg is considered as cut
off point for consideration of 100gm (W.H.O) glucose tolerance test

CRITERIA FOR DIAGNOSIS OF GDM WITH 100 GM ORAL GLUCOSE

Time mg/dl mmol/L

Fasting
95
1 hour

2 hours
180
155
3 hours
140

101
Gold standard test
ADR WORKBOOK SERIES - ENDOCRINOLOGY

Remember,
• Standard oral GTT for normal person -► 2hr test
• Standard oral GTT for pregnant women -►
3hr test
• Screening test for gestational diabetes -►
1hr in

IV Glucose tolerance is done in-

1. Children
2. Pregnancy
3.
4.
Gastrectomy
Old age
Malksorption syndrome
What is Glycosylated hemoglobin?

s Exercise
GIC HbA
Food

RBC

CII Anemia 7 Good


control
Level Micro vascular CompL
7 poor
Avg blood Glucose HbAiC 25
Serum fructosamine can be used in all except- pugo
1. Screening of diabetes
2. Rapid change in DM treatment
3. Short term control DM
4. Screening of DM in pregnancy

102
ADR WORKBOOK SERIES - ENDOCRINOLOGY
Fructosamine

fructosamine 15
se MHz 21dg
Albumin 15 2 days

CII
Management: - 1st line R LSM

exercise
iE
IFInsulin resistance
A GLUT 4 R Targettissues
30mins Brisk 5 7 Week

Oral Hypoglycaemic Agents


TIDY
Biguanides

DOC Metformin 1Insulin resistance

AfterfoodD.MIL
wt loss Never Hypoglycemia

HDAC 21
Renal failure CII
Gastritis
AE 103

Vit Brat
Lactic Acidosis
ADR WORKBOOK SERIES - ENDOCRINOLOGY

Noninsulin Antidiabetic Drugs


Class Drug Mechanism of Action Principal Adverse Drug Interaction(s)
(Target Tissue) Effects
Sulfonylureas

0
Administer 30
minutes before meals
First-Generation Enhance insulin Hypoglycemia, weight Salicylates and
Chlorpropamide secretion (beta cells) gain, hyperinsulinemia ketoconazole
(Diabinese, Insulase) increase
Acetohexamide hypoglycemia
(Dymelor)
Tolazamide
(Tolinase)
Tolbutamide
(Orinase)
Second-Generation Enhance insulin Hypoglycemia, weight Corticosteroids
Glipizide (Glucotrol, secretion (beta cells) gain, hyperinsulinemia decrease action
Glucotrol XL)
Glyburide
(Micronase, Glynase,
DiaBeta)
Glimepiride (Amaryl)
Biguanides Reduce glucose Gastrointestinal -
Administer with
meals
latter production* disturbances
(abdominal pain,
Metformin (Foramet) nausea, diarrhea),
lactic acidosis
I
Gamma-Glucosidase
Inhibitors
Delay carbohydrate
digestion (gut)
Gastrointestinal
disturbances
-

Administer just (abdominal pain, PPBG


before meals nausea, diarrhea), liver
Acarbose (Precose) function test elevation
Miglitol (Glyset)
Thiazolidinediones Improves insulin
flatulence
Headache, weight gain, -
Glitazones sensitivity (fat, flatulence,
Administer with muscle) Causes/exacerbates
meals heart failure,
Pioglitazone (Actos) decreased
Rosiglitazone hemoglobin/hematocrit
(Avandia)
Glinides Enhance insulin Hypoglycemia (less Increased risk of
Administer 15 secretion (beta cells) than sulfonylureas), hypoglycemia with
minutes before meals weight gain, salicylates,
Repaglinide hyperinsulinemia, nonselective beta
(Prandin) hypersensitivity, blockers, NSAIDs,
Nateglinide (Starlix) increased uric acid Metabolism may be
levels inhibited by azoles,
erythromycin

104
ADR WORKBOOK SERIES - ENDOCRINOLOGY
Incretin (GLP-1) Enhance insulin Gastrointestinal -
Analogues secretion (beta adverse effects
Administer 15 cells), delay gastric (nausea, vomiting,
minutes before meals emptying (gut), diarrhea)
Exenatide (Byetta) suppress prandial

SE Injected
subcutaneously
Liraglutide (Victoza)
glucagon secretion

Injected Insulin Glucagons


subcutaneously
Amylin Analogue Aids absorption of Gastrointestinal Avoid
Administer before glucose by slowing disturbances, headache anticholinergics that
meals gastric emptying alter gastrointestinal
Pramlintide (Symlin) (gut), promotes motility can delay
Injected satiety absorption of oral
subcutaneously (hypothalamic TDM Al medications;
receptors)
TDM administer oral
hypoglycemic agents
1-2 hr after Symlin
Dipeptidyl Inhibits enzymatic Runny nose, headache Hypoglycemia may

v0
Peptidase-4 breakdown of GLP-1 occur when
Inhibitors and GIP; increases combined with
Administer once daily insulin secretion; insulin or
regardless of meals decreases glucagon Peripheral edema sulfonylurea drugs
Linagliptin secretion (pancreas) Headache -

E
(Tradjenta)
Saxagliptin (Onglyza)
Sitagliptin (Januvia)
Combination Drugs
Some combination drugs include glyburide and metformin (Glucovance), glipizide and
metformin (Metaglip), and pioglitazone hydrochloride and glimepiride (Duetact).

SALT 4 56h72
2 Channel
AE V71

Dapagliflozin
PCT Glucose
E ifo3in
Ibs

CE 105
ADR WORKBOOK SERIES - ENDOCRINOLOGY
Treatment of Type 1 DM: -

Table - Insulin Preparations

Type Peak Action (Hours) Duration of Action (Hours)


Ultra-short-acting
Insulin lispro 30-60 min 4-6
Insulin aspart 20-30 min 3-5
Rapid
Regular 2-1 6-8 Longest
Semilente 2-6 10-12
Intermediate Degludec
o
NPH
Lente
6-12
6-12
12-18
12-18
48hrs
Long acting
Glargine 2 24
Levemir 18-24 36

15 30mins before
food
3 doses Regular insulin
Preferred route is: - Based

S C
I
DKAIHONK
Preferred Sites: -
Around umbilicus 2cm
away

Arm's

Thigh

Buttocks

Avoided dorsum of hand


106
ADR WORKBOOK SERIES - ENDOCRINOLOGY
Methods of Insulin delivery?

D
1. Best method is by Insulin pump

Monitor
Blood
GI
Q Inject required

at
Artificialpancreas

2. Insulin pen 
316 Needle painless 4 Bloodless
3. Insulin – Inhalation

Altvezza Exuberg

4. Multi dose vial and Syringe

Complications of Insulin usage:

107
ADR WORKBOOK SERIES - ENDOCRINOLOGY
Early morning hyperglycemia with increased blood glucose of 3.00 AM suggests –

1. Insufficient insulin
2. Dawn phenomenon
3. Somogyi effect
4. None of the above

Causes of early morning hyperglycemia in diabetics using insulin:

Causes Mechanism Diagnosis Treatment

Fasting hyperglycemia
in the early morning

1 at insulin 3AM ant dose


Insufficient
6AM
insulin
200 7 100
Dawn Phenomenon
Due to 3AM N

ming GH 6AM a

surge
nsulin
sent
Somogyi effect
(Rebound
3AM ME
Hyperglycemia) Ant dose
dope
100 200 6AM
a

108
ADR WORKBOOK SERIES - ENDOCRINOLOGY
An obese Type 2 DM patient present with FBS=180 mg% and PPBS=260 mg%
Management includes-

1. Glibenclamide
2. Diet therapy+exercise
3. Diet therapy+exercise+metformin
4. Insulin
5. Chlorpropamide

Role of insulin in type2DM-

1. Acute illness
2. Pregnancy
3. Secondary OHA failure
4. Obese patient

Insulin should be considered as the initial therapy in type 2 diabetes mellitus in


following cases –

 Insulin should be considered as the initial therapy in type 2 diabetes


mellitus in following cases –
 Lean individuals or those with severe weight loss
 Individuals with underlying renal or hepatic disease
 Individuals who are hospitalized or acutely ill
 Pregnancy
 Primary and secondary OHA failure.
 Any complication of diabetes e.g. ketoacidosis, nonketotic hyperosmolar
coma, gangrene of the extremities.
 Hyperkalemia

109
ADR WORKBOOK SERIES - ENDOCRINOLOGY
MECHANISMS OF COMPLICATIONS

Which of the following is not a advanced glycosylation end products ?

1. Pentosidine
2. Glucosepane,
3. Carboxymethyllysine
4. Methoxycellulose

I
Endothelial
injury

Most useful investigation in diagnosis of diabetic ketoacidosis -

1. Ketonemia
2. pH of blood
3. Urinary sugar
4. Urine ketone

Diabetic ketoacidosis is characterized by -


Triad

K Bodies Acidosis
250mgdA

Glucose
Acetone
Acetoacetate

hydroxy Doesnot
β butyrate
110
Most
Acidic appear urine
ADR WORKBOOK SERIES - ENDOCRINOLOGY

Pathogenesis

Insulin deficiency + Increse in counterregulatory hormone

Adipose Periphereal tissue


tissue

Activation of Hormone Increase Decrease


sensitive lipase protocolysi glucose
Liver s utilization

Increased lipolysis Amino acid


Hyperglycemia

Increased free fatty


acid

↑Fatty acid ↑Gluconeogenesis


oxidation

↑Glycogenolysis
↑Ketone body
Production

Hyperglycemia

Glucosuria+
+Osmotic
diuresis

Hyperketonemia
Dehydration

111
ADR WORKBOOK SERIES - ENDOCRINOLOGY
Which of the following statements about Diabetic Ketoacidosis is true -

1. Decreased Bicarbonate
2. Increased Lactate
3. Normal anion gap
4. Glucose <250 mg/dl

Q - C/F
Dyspnea
vomiting

polyuria
Abdominal pain

Physical findings
ARR AHR

BP fruityorder
Breath
Abdominal tenderness
Kassumal's respiration

Precipitating events: -
Cocaine Abuse severity
Mild Mod Severe
UTI
Ph 7 25 7 35 7 7 24 27.0
pneumonia
465 15 18 10 14 410
GE
112 consciousness
HAGMAD Alert Drowsy 011
ADR WORKBOOK SERIES - ENDOCRINOLOGY
Management of diabetic ketoacidosis –
KB
1ˢᵗ IVF Ns

Iv insulin Regular insulin _Intus


Antibiotics B GIC 15

Monitor Kf Hypoke
KCI.ph
27 6 4105 210 IV NOHO
A 42-year-old man with long-standing type 1 diabetes presents with gastroenteritis
that has been worsening for 5 days. His serum biochemistry values are consistent
with diabetic ketoacidosis (DKA); his blood pressure is 90/55 mm Hg, and his
heart rate is 135 beats/min. Other laboratory findings are as fol- lows: blood
glucose, 656 mg/dl; sodium, 127 mEq/L; potassium, 4.2 mEq/L; HCO3, 14 mEq/L;
anion gap, 25; and pH, 7.05. Which of the following would not be an appropriate
step in the immediate treatment of this patient?

1. I.V. administration of 0.9% saline


2. Admission to the intensive care unit
3. I.V. administration of potassium chloride
4. I.V. administration of insulin
5. I.V. administration of sodium bicarbonate

The drug of choice for managing hyperglycemia in diabetic ketoacidosis is -

1. Regular insulin
2. Lente insulin
3. Glyburide
4. 70/30 insulin

113
ADR WORKBOOK SERIES - ENDOCRINOLOGY

1. Confirm diagnosis (↑serum glucose, ↑serum p-hydroxybutyrate, metabolic acidosis).

2. Admit to hospital; intensive care setting may be necessary for frequent monitoring, if
pH <7.00, labored respiration, or impaired level of arousal.

3. Assess:

 Serum electrolytes (K+, Na+, Mg2+'. Cl-, bicarbonate, phosphate)


 Acid-base status—pH, HCO3-, PCO2 β-hydroxybutyrate
 Renal function (creatinine, urine output)

4. Replace fluids: 2-3 L of 0.9% saline or lactated Ringers over first 1-3 h (10-20 ml/kg per
hour); subsequently, 0.45% saline at 250-500 ml/h; change to 5% glucose and 0.45%
saline or lactated Ringers at 150-250 ml/h when blood glucose reaches 250 mg/dL
(13.9 mmol/l).

5. Administer short-acting regular insulin: IV (0.1 units/kg), then 0.1 units/kg per hour by
continuous IV infusion; increase two- to threefold if no response by 2-4 h. If the initial
serum potassium is <3.3 mmol/L (3.3 meq/L), do not
administer insulin until the potassium is corrected. Subcutaneous insulin may be used
in uncomplicated, mild-moderate DKA with close monitoring.

6. Assess patient: What precipitated the episode (noncompliance, infection,trauma,


pregnancy, infarction, cocaine)? Initiate appropriate workup for precipitating event
(cultures, CXR, ECG, etc.).

7. Measure blood glucose every 1-2h; measure electrolytes (especially K+,bicarbonate,


phosphate) and anion gap every 4h for first 24h.

8. Monitor blood pressure, pulse, respirations, mental status, fluid intake and output every
4h.

9. Replace K*: 10 meq/h when plasma K+ <5.0-5.2 meq/L (or 20-30 meq/L of infusion
fluid), ECG normal, urine flow and normal creatinine documented; administer 40-80
meq/h when plasma K+ <3.5 meq/L or if bicarbonate is given.
If initial serum potassium is >5.2 mmol/L (5.2 meq/L), do not supplement K+ until the
potassium is corrected.

10. See text about bicarbonate or phosphate supplementation.

11. Continue above until patient is stable, glucose goal is 8.3-11.1 mmol/L(150-200
mg/dL), and acidosis is resolved. Insulin infusion may be decreased
to 0.02-0.1 units/kg per hour.

12. Administer long-acting insulin as soon as patient is eating. Allow for a 2- to 4h overlap
in insulin and SC long-acting insulin injection.

114
ADR WORKBOOK SERIES - ENDOCRINOLOGY
Hyperosmolar nonketotic coma (HONK)

t
t.ie
R IVF
TDM GIC
the Insulin
600 1200
moldt intusion
I confusion
It Not 7150
Insulin 451.051 503m
then Ivf 12ns
Diabetic Retinopathy: -
BAC
TDM After 54
TDM Immediately Diagnosis D
CVA DNeuro
PAD D Nephro
PDRD Retina
Earliest Microaneurysms Blindness
Macular edema
Blindness R D

Nd YAG laser
R Blood GIC Control
R
In tra vitreal Blood test Monitor
Bevacizumab

progression Sr Homocystein
115
Sr creatinine
ADR WORKBOOK SERIES - ENDOCRINOLOGY
Diabetic Neuropathy: -

HbAc
Motor Autonomic
Sensory
1st

Sensation early vibration 12843

Mc form Distal symmetrical sensory neuropathy

Clf paresthesias by dysesthesias


Gabapentin pregab TCAs
R Duloxetine

Cranial MC 3rd pupillary sparing


1
Motor neuropathy Distal weakness

Autonomic neuropathy

Postural hypoth DOC Midodrine

Gastroparesis Metoclopramide

ED sildenafil
116
ADR WORKBOOK SERIES - ENDOCRINOLOGY
Diabetic Nephropathy

MCC CRF DM

DOC ACE
EI
toGFR
Earliest GFI
Microalk 30 300mg 24hrs2

CHRONIC COMPLICATIONS OF DIABETES MELLITUS

Microvascular Macrovascular

Eye disease
 Coronary artery disease
 Retinopathy (non-  Peripheral vascular disease
proliferative /  Cerebrovascular disease
proliferative)
 Macular edema
Other
Neuropathy

 Sensory and motor (mono


 Gastrointestinal
and polyneuropathy)
(gastroparesis diarrhoea)
 Genitourinary
Nephropathy
 Dermatologic
 Infections
 Cataract
 Glaucoma

117
D Amyotrophy
Strict Blood
Glucose
us
ADR WORKBOOK SERIES - ENDOCRINOLOGY
IMAGE BASED QUESTIONS

What is the use of this instrument?


Semmes Weinstei

monofilament

PN

What is this sign and in which clinical condition do you see this?
prayer's sign

D Cheiropathy

AGIse

118
ADR WORKBOOK SERIES - ENDOCRINOLOGY

Diabetic Chieropathy

PRAYER SIGN

The prayer sign is a simple clinical test that may be used to detect limited joint
mobility in the hands mobility of the patient to completely close the gaps between
opposed palms and fingers when pressing the hands together in a praying position
constitutes a positive prayer sign(A). Improvement is seen in a patient after he
received 4 months of aggressive glycemic control(B)

In which endocrine disorder do you see this?

anthelang
anthelasma

119
ADR WORKBOOK SERIES - ENDOCRINOLOGY

In which endocrine disorder do you observe this complication?

D foot

Neuropathic ulcer

sole feet
clean edges

R foot wear

off loading

Dressing
Neuropathic Ulcer Ischemic Ulcer
Painless Painful
Normal pulses Absent pulses
Regular margins, typically punched-out Irregular margin
appearance
Often located on plantar surface of Commonly located on toes, glabrous
foot margins
Presence of calluses Calluses absent or infrequent
Loss of sensation, reflexes, and Variable sensory findings
vibration
Increased in blood flow Decreased in blood flow
(atrioventricular shunting) Collapsed veins
Dilated veins Cold foot
Dry, warm foot No bony deformities
Bony deformities Pale and cyanotic in appearance
Red or hyperemic in appearance

120
ADR WORKBOOK SERIES - ENDOCRINOLOGY

What is the name of this dermatological sign and in which endocrine disorder do you
observe this?

Necrobiosis Lipoidica

diabeticorum
DM

LL

orange yellow patch


Strict Blood Gls Control

What is the name of this scan and what is its application?

Tc 99m sestamibi PT Adenomy


121
ADR WORKBOOK SERIES - ENDOCRINOLOGY

What is this sign and in which endocrine disorder do you observe this?

Grave's ophthalmopathy

NOSPECS classification
N No
eye signs a symptoms

only signs No symptoms LigLag


S soft tissue s a s stare sign

proptosis 222mm
EOM MC IR Diplopia
c corneal involvement
s sight loss ON involvement
What is the dermatological abnormality you're observing over the legs and in which
endocrine disorder do you see this?

pre tibial Myxedema


Infiltrative dermopathy

122
ADR WORKBOOK SERIES - ENDOCRINOLOGY
In which endocrine disorder do you observe clubbing of the fingers?

TÉ fÉ

Thyroid acropachy. Marked cortical thickening is demonstrated at the midshafts of


the tubular bones of the hands

Periosted cat

123
ADR WORKBOOK SERIES - ENDOCRINOLOGY

Lid retraction and proptosis in a patient with thyrotoxic Graves' disease.

Lid log sign

von Graefe's

Lagging of upper eyelid

on downward rotation

obeys
Large Multinodular Goitre

Euthyroid

HYPO

Hyper

124
ADR WORKBOOK SERIES - ENDOCRINOLOGY

Myxoedema

Round
swollenface

Swollen Lips

scanty eyebrow

Madarosis

In which endocrine disorder do you observe these dermatological changes?

purple striae

Cushing's

ACTH dependent 4M
125
ADR WORKBOOK SERIES - ENDOCRINOLOGY

In which endocrine disorder do you observe these abnormal hands?

Acromegaly

A 58-year-old man undergoes severe head trauma and develops pituitary


insufficiency. After recovery, he is placed on thyroid hormone, testosterone,
glucocorticoids, and vasopressin. On a routine visit, he questions his primary care
physician regarding potential growth hormone deficiency. All of the following are
potential signs or symptoms of growth hormone deficiency EXCEPT:

1. Abnormal lipid profile


2. Atherosclerosis GH
3. Increased bone mineral density
4. Increased waist-to-hip ratio
5. Left ventricular dysfunction

126
ADR WORKBOOK SERIES - ENDOCRINOLOGY

Following procedure is useful in the Rx of?

1. Pituitary Microadenoma
2. Pituitary Macroadenoma
3. Both
4. Hypothalamic tumors

Transphenoidal pituitary

Adenectomy

A 75-year-old man presents with development of abdominal obesity, proximal


myopathy, and skin hyperpigmentation. His laboratory evaluation shows a
hypokalemic metabolic alkalosis. Cushing syndrome is suspected. Which of the
following statements regarding this syndrome is true?

1. Basal ACTH level is likely to be low.


X
2. Circulating corticotropin-releasing hormone is likely to be elevated.
X
3. Pituitary magnetic resonance imaging (MRI) will visualize all ACTH-secreting
tumors.
X
4. Referral for urgent performance of inferior petrosal venous sampling is indicated. x
5. Serum potassium level <3.3 mmol/L is suggestive of ectopic ACTH production.

127
ADR WORKBOOK SERIES - ENDOCRINOLOGY
Name the channels on which ADH acts?

1. AQP1
2. AQP2
3. AQP3
4. AQP4

Endocrine disorder with the following presentation?

1. Acromegaly
2. Cushings syndrome
3. Addisons disease
4. Hypothyroidism

Radiograph of subperiosteal erosions (lower arrows) in a phalanx with terminal


resorption (top arrow) in a patient with primary hyperparathyroidism

Hyperparathyroidism

128
ADR WORKBOOK SERIES - ENDOCRINOLOGY
Condition seen in.

1. MEN 1
2. MEN 2a
3. MEN 2b
4. MEN 4

Neurone

DIABETES INSIPIDUS

Central DI – Etiology
Head injury
Nephrogenic DI – Etiology
Drugs Li AG Amph B Cisplatine MF D
Clinical Features
Polyuria Polydipsia
Investigations
MD a sv.osm.fur.am treat
DOC for Central DI
Desmopressin
DOC for Nephrogenic DI

DOC for Lithium DI Hydrochlorthiazide


D/D for DI
Amiloride
psychogenic polydipsia
Sv Osm
Ur 05m

129
ADR WORKBOOK SERIES - ENDOCRINOLOGY
SIADH

Compensatory Mechanisms Franef


SNS
Etiology – Malignancy
Small cell Ca Lung
Etiology – Drugs & Neurology
TCAs SSRIs Desmopressin chlopropamine
Clinical Features
CIF f Nat BP N No slo fluid
Investigations
Isr 05m Aur Osm Auv Not test Not
IOC

1st line treatment


water loading
DOC
fluid
VAPTANS

PROLACTINOMA
MC secretory
MICRO & MACROADENOMA
10mm 10mm
Physiological

Pathological
preg Lactation sleep stress Nipple 4
Tumors Systemic Bond Drugs
Clinical features – Females
Galactorrhea Amenorrhea infertility
Male

Mass effect
Libido Infertility
IOC Bitemporal hemianopia
1st line treatment
Sv pre 5 20mg d
Dopamine t

130

TPPE Sx
ADR WORKBOOK SERIES - ENDOCRINOLOGY
ACROMEGALY

Macro Microadenoma

Skeletal manifestations death


Extremities Face Arthritis
Soft tissue manifestations
Skin Tongue Morgans CHF
CNS manifestations

Abnormal metabolisms
Hache Diplopia
1 line investigation
st
Glucose Dyslipidemia Galactorth

IOC
IGF 14
GH test
1st line treatment

Drug of choice
surgical resection
octreotide
HYPOPITUTARISM
2nd PEGU SOMALI
1ST Hormone reduced
GH
MCC

Pituitary Apoplexy pituitary Adenoma


Inflammatory causes Hemorrhagic Int P Adenoma
Clinical features
Granulomatous intl
ACTH levels
Hormone

GH levels Metyrapone test


Other hormones Arginine
Drug of choice
TSH LH FSH
41st Hydrocortisone
131
ADR WORKBOOK SERIES - ENDOCRINOLOGY
CUSHINGS SYNDROME / CUSHINGS DISEASE

MCC MC non latrogenic & ACTH dependent -


Iatrogenic PAdenoma
MC ACTH independent
A Adenoma
Earliest C/F
wt Gain
Dermatological
5 Atrophy Bruise Striae Hyperpig
Metabolism
Glucose AFFA prox Myopathy
CVS & Neurology
HTN Psychosis
1st Line investigations

IOC of ACTH dependent


24hr urinary cortisol
MRI N High dose Dx
1 line treatment -
st
DOC -
Sx ketoconazole
CONNS SYNDROME

MCC hyperaldosteronism
Le BK Adrenal hyperplasia
MCC Conns syndrome

Clinical features
Adrenal Adenoma
D Here No edema polyuria Muscle
Aldosterone escape phenom.

1st line Inv./Screening test


No edema Met Alkalosis
IOC ARE
1st line treatment
Oral salt solution Iv saline to
If B/L adrenal tumors
surgical resection
D/D – Liddle’s syndrome
spironolactone
AD
ENAC
132

DOC Amiloride
ADR WORKBOOK SERIES - ENDOCRINOLOGY
ADDISONS DISEASE – Primary/Secondary

MCC - India MCC – West


TB Autoimmune Adrenalitis
MC virus MC Fungal -
CMV HIV Histoplasmosis
Adrenomyeloleukodystrophy
Linked
Waterhouse Friederichsen
N meningitidis Acute
Steroid deficiency
wt loss Hypoglycemia HYPOTN
Aldosterone deficiency
Not postural Hypothe salt 4kt
Weak sex hormone deficiency
Libido Loss SCC
Secondary - Pigmentation
Ie palms a sole
1
Alabaster

ADDISONS DISEASE – Primary/Secondary

Primary Secondary

Steroid SxH Ald Steroid G SxH Ald N


ACTH A MSHA
IOC
ACTH MSHI
DOC for Primary
co syntopin test

DOC for Secondary


Hydrocortisone oral
Doc for WFS Dexa oval
Hydro IV
ceftriaxone
IVF NS
133

Dextrose
ADR WORKBOOK SERIES - ENDOCRINOLOGY
PHEOCROMOCYTOMA

Most predominant hormone

Extra Adrenal Pheo.


ME
MC symptom & sign
organ of ZuckerKandle NE
H A G HTN
Carbohydrate Metabolism
Glucose
Haematocrit & Calcium
A
Postural reflexes
Postural Hypothe
1st Line inv. & IOC

Imaging of choice
24hr Urinary catechol Pl Meta 974
6968 DOTATATE PET
TOC

DOC
Sx
Phenoxybenzamine Auerbach
HYPERPARATHYROIDISM

MCC PRIMARY HYPERPTH

SECONDARY CAUSES
Solitary PT Adenoma
RF Vit Dt Malabs Bisphol
C/F:- HYPERCALCEMIA

BONE ABNORMALITIES
Renal stone 2 parkin
Osteoporosis pinhead stippling
cod fish RuggerJersey Loss of L dura
CAUSE OF DEATH

INVESTIGATIONS
systolic arrest
Cat PAPTHAAIKPO4 DE A
IMAGING – LOCALISATION

TREATMENT
TC 9am sestamibi
surgical
134
resection
Cat IVF furosemide
ADR WORKBOOK SERIES - ENDOCRINOLOGY
HYPOPARATHYROIDISM

ETIOLOGY

DIGEORGE SYNDROME
Al I Mgt Thyroid Sx
C/F: - HYPOCALCEMIA
Chr 22
perioral a periungual paresthesias
Tetany Troussenau he chorsteke's
G vorum
CAUSE OF DEATH
G valgum Short stature Delayed Dentiti
INVESTIGATIONS Laryngo spasm
TREATMENT
PTH Cat Ap A QT int
Acute IV Chronic oral
PSEUDO HYPOPARATHYROIDISM
GMAS GSX subunit t
R oral Cath
HYPERTHYROIDISM & HYPOTHYROIDISM

GRAVES HASHIMOTOS THYROIDITIS


MCC MCC
TSI TPO Ab's
C/F :-
Heat intoli cutloss cold intol wt Gain
Tachy CHF Here Brady D Here P Effusion
AGIC Proxmyopathy Dyslipidemia IRF
ARR Diarrhea Tremors constipation Hung up reflex
ophthal p myxedema carotenemia
JODBASE DOW WOLF CHAIKOFF

Hyper T Hypo
135 E organification
ADR WORKBOOK SERIES - ENDOCRINOLOGY
HYPERTHYROIDISM & HYPOTHYROIDISM

INVESTIGATIONS
All A Hyper 21 Hypo Aed All
7374 A TSHI 1 Hyper 7374 TSHA Ie Hypo
RAIU:
11321 I 123

Rx. :-
Graves Dequervains
DOC Methimasole DOC LTG 16mg kg
preg 1st PTU M coma IV LT4
2nd 930 Methimazole IV LT
DOC T Storm NG PTU
MEN 1 MEN 2a MEN 2b/3 MEN 4

OTHER NAME

CHROMOSOME

GENE

TUMORS

EXTRA

ENDOCRINE

136
ADR WORKBOOK SERIES - ENDOCRINOLOGY
DIABETES & COMPLICATIONS

TYPE 1 DM TYPE 2 DM

PATHOGENESIS
B cell Insulin resistance
INSULIN
Nla
AGE & BMI
2220
HLA ASSOCIATION
N 30 red
DRB P4DQ2 NO
C/F
P P P
COMPLICATIONS
Rare AIntections
DKA TST HONK 72771
MICRO VASCULAR
D Retino D Nephro D Melero
MACRO VASCULAR
CAD PAD
WAI

INVESTIGATIONS – DIABETES MELLITUS

NORMAL IMPAIRED DM RANGE TARGET

FBS
4100 101 125 3126 2130
PPBS
4140 141 199 7200 2180
HBA1C
256 5.7 6 4 76.5 47
GOLD STD.
OGTT

137
ADR WORKBOOK SERIES - ENDOCRINOLOGY
CAUSES OF EARLY MORNING HYPERGLYCEMIA

Mechanism Diagnosis Treatment

Fasting
3AM Insulin
Hyperglycaemia Insuff
Dawn phenomenon
Insulin 6AM
3AM N GAMA
Somogyi effect
3AM GAMA
49
TREATMENT OF DIABETES MELLITUS
1st line LSM
TYPE 1 DIABETES MELLITUS TYPE 2 DIABETES MELLITUS

Insulin OHA Metformin

MATURITY ONSET DIABETES OF YOUNG

PATHOGENESIS INHERITANCE BMI & TYPES & TREATMENT


COMPLICATIONS GENE
MUTATION

B cell dysf 2 Generation Low dose


a Gc
Never DKA
LATENT AUTOIMMUNE DIABETES OF ADULTS
µµf
PATHOGENESIS ANTIBODIES AGE TREATMENT

Autoimmunity Anti GAD Aduts Insulin


ICA
138
ADR WORKBOOK SERIES - ENDOCRINOLOGY

Carcinoid syndrome

C Tumor slow growing tumor of Neuroendocrine


tumour
Mid gut Fore gut

Jejunum ileum cecum


Appendix

X metabolites of Tryptophan serotonin


5H AA 54T 5H

mine Phat
Typical Atypical

Serotonin N
serotonin
5HTP
urinary 5HAA
fore gut
Midgut
ve
Argentattine ve

5HIAA A
A 45hL
139
ADR WORKBOOK SERIES - ENDOCRINOLOGY

t.IT
CVS
Digha Hepatomegaly Broncho
Flushing
constriction
Abd
Head paid Wheeze
Upper thorax

PSMM
T insufficiency TPE
p stenosis
EG
A Localization Octre0

to
60,68 Dotatate PET
861

B octreotide Tumors Localised


retractory
surgical B
O Telotristate
140
Mets CT 5FU 6

Doxorubicin
ADR WORKBOOK SERIES - ENDOCRINOLOGY

141
ADR WORKBOOK SERIES - ENDOCRINOLOGY

142

You might also like